You are on page 1of 77

Star Rating

On the basis of Maximum marks from a chapter


On the basis of Questions included every year from a chapter
On the basis of Compulsory questions from a chapter

Nil
jjjj
Nil

Information System Concepts

This Chapter Includes : Introduction; Definition of a System; System Environment;


Types of Systems; Sub-systems; Information; Business Information Systems.
Marks of Short Notes, Distinguish Between, Descriptive & Practical Questions

Q&A-6.15

Q&A-6.16

Solved Scanner

CA Final Gr. II Paper - 6

CA Final Gr. II
S HORT N OTES
2001 - Nov [7] Write short notes on the following :
(i) Programmed and non-programmed decisions.
(5 marks)
Answer :
Organisational decisions can be categorised as programmed decision & non
programmed decision.
Programmed decisions: Decisions which are of repetitive and routine nature are
known as programmed decisions. It is also known as structured decisions. It refers to
decisions made on problems and situations by reference to as predetermined. Set of
precedents, procedures, techniques and rules.
e.g. preparation of creditors due list and disbursement of its payments through
bank account. For such type of decision making. Guidelines and rules required are
provided in the form of a procedure manual.
Non-programmed decisions : Decisions which are unstructured involve high
consequence, complex or a major commitment are known as non-programmed
decisions, Non programmed decision are those decision, which are made on situations
and problems that are novel and non-repetitive. For example, new product line; capital
budgeting etc. Non-programmed decisions making has no pre-established decision
procedure. Also, it is difficult to completely specify the information requirements for
taking these decisions.
Information requirement depends upon type of decision making. In the case of
programmed decision, the decision is pre-specified. Such a decision clearly specifies
the information requirements, for example, in the case of an inventory item, the reorder
level is fixed. For placing a fresh order for inventory replenishment, one requires
information about its present level.
2002 - Nov [7] Write short notes on the following :
(c) Decision Support systems
(5 marks)
Answer :
Decision support system (DSS) : Decision support system aims at assisting
managers, who are faced with unique and non-recurring decision problems. In such
cases, it is not possible to pre-design system report format and contents. Hence, a DSS
has greater flexibility than other information system.

[Chapter # 1] Inform ation System Concepts O

Q&A-6.17

DSS is of more use when the decision are of an unstructured (i.e. for strategic
decision) or semi-structured nature (i.e. for tactical decision) .
The problem area is modeled and various alternatives are explored, e.g. an Excel
Spreadsheet is a form of DSS.
C
All Information Systems support decision-making, however, indirectly, but DSS are
expressly designed for this purpose.
C
DSS provides its users with decision oriented information whenever a decisionmaking situation arises.
C
It is a computer-based Information System specifically designed to assist
managers in making semi-structured/unstructured decisions, (i.e. those
decision-making situations that cannot be predicted in advance).
C
It offers interactive display technology, micro- computing easy-to-use software that
provides support during the decision-making process.
C
A DSS does not typically make decisions or solve problems - people do, DSSs are
concerned with providing useful information to support the decision process.
C
It is typically used by top level managem ent.
Characteristics of DSS
The DSS are characterised by at least three properties nam ely;
1. Ability to support semi-structured and unstructured decision-making-structured and
semi-structured are those that are easily made from a given set of inputs. They are
of the routine nature. But some semi-structured and all unstructured decisions
require information from m any sources to sum up the data and to take a decision.
The DSS is particularly well adopted to help with semi-structured and unstructured
decisions.
2. Offer users flexibility, adaptability and a quick response - DSS are more suitable
for semi-structured/unstructured decisions which are not pre-planned or prespecified and tend to be non-routine. Here, DSS must offer enough flexibility to
enable users to model their own info needs.
3. Ease of learning and use -Since DSS are often built and operated by managers
(i.e. the users) rather than by computer professional. Therefore, the accompanying
tools/models should be easy to learn and use. If requires the use of
1. Graphical User Interfaces (GUI); and
2. Interactive display devices - user of interactive display devices are not
compulsory to run a DSS, their use makes the system user-friendly and
effective.
2003 - May [7] Write short notes on the following :
(d) Materials Requirement Planning (MRP)

(5 marks)

Q&A-6.18

Solved Scanner

CA Final Gr. II Paper - 6

Answer :
Material Requirement Planning (M RP) System :
Scope : W hen the Company is following a fixed period production programming to
maximise the use of production capacity, the Material Requirement Planning systems
are used to minimise the inventory and simultaneously ensure the availability of the
material.
Applicability : The MRP system is feasible only when the company produces standard
products for which the bill of Material or the Part List enumerating the items which go
into the products, is available.
The bill of material is exploded and summarised for the items disregarding where
it is used. Then it is compared with the stock available, the order placed and due for
delivery in the production period. The comparison will show the shortage which would
be faced in the execution of the production programme.
MRP system is also used when the production requirement is fluctuating from
period to period and also where the changes in the production programme are frequent
use as
Use as a tool : It provides an excellent tool for planning, procurement, monitoring and
control of the inventory as well as the production programme.
2003 - Nov [7] Write short notes on the following :
(a) Executive Information Systems.
(5 marks)
Answer
Executive Information System is an information system that serves the information
need of top executives. EIS enables its users to extract summary data and model
complex problems without the need to learn complex query languages, statistical
formulas or high computing skills.
EIS is considered as highly user friendly system because it provides a user friendly
graphical reporting system with drill down capabilities. EIS is mainly an advancement
of MIS but it can include the decision support system (DSS) capabilities to solve
complex problems.
Executive Information System (EIS) Vs Executive Support System (ESS): An
executive information system (EIS) is sometimes referred to as an executive support
system (ESS)- is a DSS that is designed to meet the special needs of top-level
managers. Some people use the term s EIS and ESS inter changeably, but others
do not. Any distinction between the two usually is because executive support systems
are likely to incorporate additional capabilities such as electronic mail.

[Chapter # 1] Inform ation System Concepts O

Q&A-6.19

Advantage of an EIS :
1. Easy Access: Executives can have easy access to corporate data. DSS provides
executives with access to Financial data Marketing and Sales information, Human
Resources information, Manufacturing data and Competitive / Strategic
information. Electronic mail, access to external news and databases, word
processing, Spreadsheet and automated filing capabilities are also common in
business EIS.
2. Easy Navigation : An EIS is easy to navigate since Managers can identify broad
strategic issues, and then explore the information to find the root causes of those
issues.
3. Report Generation : Reports can be generated by the Executives directly, who
can now examine departments without interfering with the operations of the
Company. Graphs can be created automatically.
4. Simulation Facility : Executives can set up different scenarios or simulations. The
ability to manipulate data, to project what if outcomes and to work with modeling
tools within the system are available in EIS.
5. Cost vs. Benefits : EIS is expensive to develop and maintain. But enhanced toplevel decision making is the benefit that balances out the systems associated
costs.
2004 - May [7] Write short notes on the following :
(d) Expert systems.
(5 marks)
Answer :
Expert System (ES) : An expert system, also called a knowledge-based system, is an
artificial intelligence system that applies reasoning capabilities to reach a conclusion.
Expert systems are excellent for diagnostic and prescriptive problems. Diagnostic
problems are those requiring an answer to the question, "Whats wrong?" and

Q&A-6.20

Solved Scanner

CA Final Gr. II Paper - 6

correspond to the intelligence phase of decision making. Prescriptive problems are


those that require an answer to the question, "What to do?" and correspond to the
choice phase of decision making.
An expert system is usually built for a specific application area called a domain.

Expert Systems are an extension of DSS/EIS.

It is a computer program that uses artificial intelligence to solve problems within


a specialized domain that ordinarily requires human expertise.

They mimic the process of human expert for those who has less expertise.

These are rule-based and pre-programmed on certain assumptions and


criteria.

They use static logic to solve problems.

Typical tasks for expert systems involve classification, diagnosis, monitoring,


design, scheduling, and planning for specialized endeavours

Areas covered by expert systems include; Audit, Tax, Financial Services,


Manufacturing, Retailing, and Logistics etc.

Nevertheless, Expert Systems remain supplements rather than replacements


for human experts.
Need for Expert Systems : Expert labor is expensive and scarce, moreover, no matter
how bright or knowledgeable certain people are, they often can handle only a few
factors at a time.
Benefits of Expert Systems :
1. Expert Systems are not subject to such human fallings as fatigue, being too busy,
or being emotional.
2. Expert Systems assist novices in thinking the way experienced professional do.
3. Expert System s preserve knowledge that might be lost through retirement/
resignation/ death.
4. Expert Systems put information into an active-form so it can be summ oned almost
as a real- life expert might be summoned.
5. Expert Systems can be effectively used as a strategic tool is the areas of marketing
products, cutting costs and improving products.
Some of the properties to qualify for Expert System development are as under :
(i) Availability : One or more experts are capable of communicating how they go
about solving the problems to which the Expert System will be applied.
(ii) Complexity : Solution of the problems requires logical inference processing,
which would not be easily handled by conventional information processing.
(iii) Domain : The domain or subject area, of the problem is relatively small and
limited to a relatively well- defined problem area.
(iv) Expertise : Solutions to the problem require the efforts of experts.
(v) Structure : The solution process must be able to cope with ill-structured,
uncertain, missing, it conflicting data, and a dynamic problem-solving situation.

[Chapter # 1] Inform ation System Concepts O

Q&A-6.21

Components of Experts Systems :


An Expert System provides tools, information and m ethods for decision making in
specific areas such as generating competitive bids, loan approved and so on.
An Expert System is typically composed of following elements:

Knowledge Base is an organized collection of facts about the systems domain.

Inference Engine interprets and evaluates the facts in the knowledge base in
order to provide an answer the Knowledge Base, Inference Engine, the knowledge
Acquisition subsystem and the user interface.

Knowledge Acquisition Subsystem (KAS) is the software component of an


Expert System that enables the Knowledge Engineer (KE) to build and refine an
expert systems knowledge base. The KE works with the knowledge acquisition
subsystem to model decision logic.

User Interface is the method by which an expert system interacts with a user.
These can be through dialog boxes, command prompts forms, or other input
methods. The user interface for an Expert System is highly interactive.
2004 - Nov [7] Write short note on the following :
(a) Strategic and Tactical decisions
(5 marks)
Answer :
Strategic Decisions :

Strategic Decisions are the decisions made at the strategic level of the organization
to handle problems critical to its survival and success

They have a vital impact on the direction and functioning of the organization.

Such decisions are taken by Top management / Board of Directors

Strategic decisions are made under conditions of partial knowledge and generally
un-structured and can not be programmed . It requires qualitative & quantitative
information.

Typical responsibilities under strategic level decision making are : Defining


objectives, policies for whole organisation, long term planning, Large scale
investment decisions, Appointment of middle management etc.

Strategic decision requires external information like market and economic


forecasts, political and social trends legislative technological constrains and
opportunities.

Strategic decision are taken by top level managem ent

For which they uses Decision Support System (DSS) and Executive Information
System (EIS).

Under strategic decision human involvement is essential and application of


computer is required for storage and analysis of various data.

Q&A-6.22

Solved Scanner

CA Final Gr. II Paper - 6

Tactical Decision :
Tactical level decisions are to be taken by middle level of managerial hierarchy. At this
level, managers plan, organize, lead and control the activities of other managers. A
single strategic decisions calls for a series of tactical decisions, which are of a relatively
structured nature.

Tactical decisions are relatively short, step-like spot solutions to breakdown


strategic decisions into implementable packages.

Such decisions are semi-structured. The other features of tactical decisions are;
they are more specific and functional they are made in a relatively closed setting;
information for tactical decisions is more easily available and digestible; they are
less surrounded by uncertainty and complexity; decisions variables can be
forecasted and quantified without much difficulty and their impact is relatively
localized and short-range.

Tactical decisions are made with a strategic focus.

Typical responsibilities of tactical decisions are establishment and monitoring of


budgets, Acquisition of resources, developing operational policies and objectives,
appointment of staff etc.

Tactical decisions requires external informations like cost and sales analysis,
performance measurement, summaries of operations / production, actual and
budgeted comparison etc.

Tactical decisions are taken by middle level management for which they use
management inform ation system (MIS).

Some examples of such decisions are taken by middle level management for
which they use management information system (MIS).

Some examples of such decisions are pricing, capacity planning, budget


preparation, purchasing contracts etc.
Thin line Distinction :- As in the case of programmed and non-programmed decisions,
the dividing line between strategic and tactical decision is thin. e.g. Product pricing is
tactical decision in relation to the strategic decision of design and introduction of a new
product in the market. But product pricing appears to be a strategic decision to downline tactical decisions on dealer discounts.
2006 - May [7] (a) Write short notes on the following :
(iii) Material requirement planning.
Answer :
Please refer 2003 - May [7] (d) on page no. 17
2008 - May [7] Write short notes on the following :
(a) Closed and open systems
(b) Programmed decisions

(5 marks)

(5 marks)
(5 marks)

[Chapter # 1] Inform ation System Concepts O

Q&A-6.23

Answer :
Closed system :

A closed system is one which does not interact with its environment. Such systems
in business World, are rare, but relatively closed systems are common. Thus, the
systems that are relatively isolated from the environment but not completely closed,
are termed closed systems (which actually means a relatively closed system ).

Example of close system : A computer program is a relatively closed systems


because it accepts and processes previously defined inputs and provides outputs too
in a previously defined way. In other words, we may say that a relatively closed system
is one which controls its inputs. And outputs and is protected from the environmental
disturbances.

In organisations and in information processing, there are systems that re relatively


isolated from the environment but not completely closed in the physics sense.
These will be termed closed systems, meaning relatively closed.

Example of relatively closed system : Systems in manufacturing are often


designed to minimize unwanted exchanges with the environment outside the
system. Such systems are designed to be as closed as possible, so the
manufacturing process can operate without disturbances from suppliers,
customers, etc. A computer program is a relatively closed system because it
accepts only previously defined inputs, processes them, and provides previously
defined outputs.

In summary, the relatively closed system one that has only controlled and well
defined inputs and outputs. It is not subject to disturbances from outside the
system.
Open System :

Open System s actively interact with other systems and establish excange
relationship. They exchange information, material or energy with the environment
including random and undefined inputs.

Q&A-6.24

Solved Scanner

CA Final Gr. II Paper - 6

Open systems tend to have form and structure to allow them to adapt to changes
in their external environment for survival and growth.

Organisations are considered to be relatively open systems. They continuously


interact with the external environment, by processes or transformation of inputs into
useful output.
However, organisation behave as a relatively closed system in certain respects so
as to preserve their identity and autonomy, They may ignore many opportunities
so as to maintain their core-competence.
Organisations are open systems, because they are input-output systems. The input
consists of finance, physical & mental labour and raw material.
Open systems are adaptive in nature as they tend to react with the environment in
such a way, so as to favour their continued existence. Such system s are selforganising, in the sense that they change their organisation in response to
changing conditions. All living systems (e.g) humans, plants, cells, etc.) are open
systems.
They attempt to maintain equilibrium by homeostasis, the process of adjusting to
keep the system operating within prescribed limits.
An organisation which is sensitive to changes in customer tastes, preferences,
likings, dislikings, demands, etc., and in consequence adjusts its prices, changes
its product mix or looks for new markets, is an open organisation.
All organisations essentially are open systems as they cannot work in isolation.
Thus, the systems analyst usually deals with adaptive, open systems.

(b) Programmed Decisions :


Meaning - If refers to decision-making situations that can be predicted in advance.
These are well-defined decisions. As a problem or issue for decision-making
emerges, the decision is made by reference to pre-decided; Guidelines,
Precedents, Rules, Standard Operating Procedures.
These are time-tested for their validity.
Example :Resource Scheduling, Inventory Control Decision, Machine Loading and
so on.

Advance knowledge of decision-making situations - These are known in


advance.

Characteristics- Routine, repetitive, often automated.

[Chapter # 1] Inform ation System Concepts O

Q&A-6.25

Involvement of thing /people-usually involve thing (i.e. Computer resources)


rather than people.
Delegation to low-level management- it is possible.

D ISTINGUISH B ETWEEN
2007 - May [6] (c) Differentiate between open and closed systems.
(5 marks)
Answer :
An open system is one that interacts with its environment and thus exchange
information, material or energy with the environment, including random and undefined
inputs. Open systems are adaptive in nature as they tend to react with the environment
in such a way, so as to favour their continued existence. Such systems are selforganising, in the sense that they change their organisation in response to changing
conditions. All living systems such as humans, plants cells, etc. are open system.
W hereas, a closed system in one which does not interact with its environment.
Such systems in business world, are rare, but relatively closed systems are common.
Thus, the systems that are relatively isolated from the environment but not completely
closed, are termed closed system (which actually means a relatively closed system). No
system can be regarded as completely closed, since it will have certain properties of
open systems also. One has to view systems in terms of the degree to which it is open
or close. The difference between open system and close systems are :
Particulars

Closed Systems

Open System

1. Meaning

A Closed System is self- Open Systems actively interact


contained and isolated from its with other systems and establish
environment.
exchange relationship with the
environment.

2. Exchange

It does not interact or make any


exchange with its environment.
A relatively closed system may
accept defined inputs, but does
not react to disturbances, i.e.
undefined inputs.

3. Feedback

They do not get the feedback Open Systems adapt to changes


from the environment required based on feedback from their
to sustain them.
external environment for their
survival and growth.

T h e y e x c h a n g e r e so u r c e s,
information, material or energy with
the environment including random
and undefined inputs.

Q&A-6.26
4. Continuity

Solved Scanner

CA Final Gr. II Paper - 6

They tend to deteriorate or They sustain themselves over a


vanish over time. Life Cycle of a period of time, by generating
Closed System is shorter, surplus out of its processes.
because it does not have any
input/ interaction from environment, and decays faster.

D ESCRIPTIVE Q UESTIONS
1999 - May [1] {C} (a) Discuss the characteristics of an effective Management
Information System.
(10 marks)
Answer :
Management Information System (MIS) :
Meaning & Concept :

Management Information System (MIS) has been defined by Davis and Olson as
an integrated user-machine system designed for providing information to support
operational control, management control and decision making functions in an
organization.

MIS produces prescribed reports, displays, and responses on a periodic,


exception, or demand basis to managers at all levels in all functions in an
organization.

MIS is typically used by Middle Level Managers.

MIS analyses the data in predetermined manner.

MIS is a network of information that supports managem ent decision - making. It


recognises information as a resource, and than uses it for achievement of
organisational objectives.
Characteristics of an effective MIS :
1. Heavy planning element

Making MIS is not a 1 or 2 day exercise. It Usually takes 3 to 5 years and


sometimes a much longer period hence future objectives have to be kept in
mind, so that MIS will not be obsolete before it gets into action.
2. System Approach

The information system follows a Systems approach. The systems approach


implies a holistic approach to the study of system and its performance in the
light of the objective for which it has been constituted.

This approach is anti- piecemeal in nature.

[Chapter # 1] Inform ation System Concepts O

3.

4.

5.

6.

8.

Q&A-6.27

In other words, systems approach, in the sense intended here, means taking
a comprehensive view or a complete look at the interlocking sub-systems that
operate within an organisation.
Sub System Concept

Although MIS is viewed as single entity, but for its effective use it should be
broken down in small parts or subsystems, so that more insight and attention
is paid to each

Sub system
Management Oriented

A good MIS must furnish information to the managers to expand their


knowledge base.

For designing of MIS, top-down approach should be followed. Top-down


approach suggests that the system development starts from the determination
of managem ent needs and overall business objectives.

The MIS development plan should be derived from the overall business plan.

Management-oriented characteristic of MIS also implies that the management


actively directs the system development efforts.

In MIS development, a manager should spend a good amount of his/her time


in system design.

To ensure that the implemented system meets the specifications of the


system, continued review and participation of the manager is necessary.
Need Based

MIS design and development should be as per the information needs of


managers at different levels,

In other words, MIS should cater to the specific needs of managers in an


organisations hierarchy.
Management directed MIS is meant for managerial decisions hence manager
should guide the MIS professionals about their information needs.

User-friendly MIS should be such that can be used by the end-user so that
they do not depend on the experts.
Integrated

Integration is a necessary characteristic of a management information system.


Integration is significant because of its ability to produce more meaningful
information.

MIS integrates various subsystem to provide for meaningful information. For


example, in order to develop an effective production scheduling system, it is
necessary to balance such factors as:
(i) set-up costs,
(ii) workforce,

Q&A-6.28

9.

10.

11.
12.
13.

14.

15.

16.

Solved Scanner

CA Final Gr. II Paper - 6

(iii) overtime rates,


(iv) production capacity,
(v) inventory level,
(vi) capital requirements,
(vii) customer services, etc.
Exception Based

MIS should be developed on the exception-based reporting principle, which


means an abnormal situation, i.e. the maximum, minimum or expected values
vary beyond tolerance limits.

In such situations, there should be exception reporting to the decision-maker


at the required level.
Future Oriented

Besides exception-based reporting, MIS should also look at the future.

MIS should not merely provide past or historical. Information; rather it should
provide information, on the basis of projections based on which actions may
be initiated.
Flexibility There should be room for further modification.
Information as a resource Information is the major ingredient of MIS, so it should
be treated as a source and managed properly.
Computerized We can use MIS without computer. But the use of computers
increases the effectiveness and efficiency of the system. The other benefits are
accuracy, storage capacity and timely information.
Common data base

To be cost-effective, MIS should avoid duplication of files and must have


common database.

It seems logical to gather data once, to properly validate it and to place it on


a central storage medium, that can be accessed by any other sub-system.
Common data flow

By common data flows, we mean to use common input, processing and output
procedures.

The common data flow concept supports several of the basic tenets of system
analysis.

These include avoiding duplication, combining similar functions and


simplifying operations wherever possible. The development of common data,
flow is an economically sound and logical concept, but it must be viewed in a
practical light.
Long-Term Planning

MIS is developed over relatively long periods. Such systems do not develop
overnight. A heavy element of planning is involved.

[Chapter # 1] Inform ation System Concepts O

Q&A-6.29

The MIS designer must have the future objectives and needs of the company
in mind. The designer must avoid the possibility of the system going obsolete
before its time.
17. Information relevance: To prevent information overload MIS should aim at
providing only relevant information.
1999 - Nov [1] {C} (a) What are the main pre-requisites of an effective MIS? Explain
them briefly.
(10 marks)
Answer :
The main pre-requisites of an effective MIS are as follows :
1. Qualified system and management staff

In a MIS, we have two types of experts: one computer and system expert
and other managements staffs who direct the system experts about their
needs and requirements from a MIS. Both of them should have expertise in
their respective fields.

But they must possess certain knowledge of other fields also. E.g. system
expert must know management consents. Same is the case with management
experts.

They must have some basic knowledge of computers so that they can help in
system designing and independently use the system.
2. Active participation of operating management
After implementation of MIS operative staff may show some behavioural problem,
which can be handled by educating the operative management about the benefit
of MIS.
3. Top management support

To be effective, MIS requires full support of top management if it is absent


then subordinates become lethargic and disinterested.

To gain the support from top m anagem ent, it should be provided with cost
benefit analysis of having MIS.

This action of subordinates will lead to change in the attitude of top


management towards MIS and theyll give their full support.
4. Database
C
It is a super file which has all records or data to a particular organisation at
one place.
C
It can be subdivided into various subsets namely; employee file, inventory file,
General Ledger accounting file, supplier file, customers file etc. Database
should be user oriented.
C
It should be controlled by separate authority that is the DBA (database
administrator).

Q&A-6.30
5.

6.

Solved Scanner

CA Final Gr. II Paper - 6

Control and Maintenance of M IS

Control refers to the process of ensuring that MIS is operating as it was


designed to operate. Sometimes users develop their own procedures or
shortcuts to use the system that may reduce its effectiveness.

To check such practices, the management at each level should devise check
mechanism for information system control.

Maintenance means alterations and modifications according to the need and


requirements of the user. So there must be some scope of improvement in the
system.
Evaluation of MIS : To meet the future needs MIS should be evaluated at regular
time intervals. Evaluation process must have following steps:

Evaluate whether system is flexible enough to cope with any expected or


unseen information requirements in future.

Take the feedback from users as well as designers about capabilities and
shortcomings in the system.

Guide the proper authority to maintain the effectiveness of MIS.

1999 - Nov [2] (a) What are main objectives of a production scheduling department?
(5 marks)
Answer :
Objectives of a production scheduling department
The main objectives of production scheduling departments are :(i) To determine the stages of production in sequential and rational order.
(ii) To assess the extent of need of sub-contracting to outside parties.
(iii) To minimize the idle time on the part of the operators and equipments.
(iv) To ensure that completion dates or targets dates of completing the production
plans are m et fully.
(v) To study alternative methods of performing the activities so that time taken to
perform can be further reduced.
2000 - Nov [1] {C} (a) Define Personnel Information system. W hat sub-system s are
generally responsible to control the operational efficiency of the Personnel
Management? Explain each one of them.
(10 marks)
Answer :
Personnel Information System

The personnel information system deals with the flow of information about people
working in the organisation as well as future personnel needs.

In most of the organisations, the system is concerned primarily with the six basic
sub-systems of the personnel function; recruitment, placement, training and
development compensation, maintenance and health and safety.

[Chapter # 1] Inform ation System Concepts O

Q&A-6.31

It is generally accepted that the personnel function is one of the least computerised
of all the functions.
Automated system may not be necessary for small, but large business firms are
realising that com puter based personnel information systems are necessary for
increasing the operational efficiency of personnel management.
Basically, many organisations used computer-based information systems to
produce pay cheques and payroll reports, maintain personnel records and analyse
the use of personnel in business operations.
Many organisations have gone beyond these traditional functions and have
developed personnel information systems, which also support (i) recruitment,
selection and hiring (ii) job placement, (iii) performance appraisals, (iv) employee
benefit analysis, (v) training and development and (vi) health, safety and security.
Such information system support the concept of human resource management
(HRM). The effective and efficient use of the human resources of a company.
The personnel system should be organised on functional basis. It should have the
following information sub-systems to increase the operational efficiency of
personnel managem ent.
1. Recruitment :
C
Properly managed recruitment sub-system may forecast personnel needs
and skills required for recruiting personnel at the proper time to meet
organisational manpower needs.
C
Such a sub-system may not only furnish information concerning skills
required for company programmes and processes but also maintains the
inventory of skills available within the organisation.
2. Placement :
C
This sub-system is concerned with the task of m atching the available
persons with the requirements.
C
A good placement sub-system makes use of latest behavioral tools and
techniques.
C
It ensures that the capabilities of people are identified before being
matched with properly organised work requirements.
3. Training and development :

As technological changes and demands for new skills accelerate, many


companies find that they must develop much of their requirements from
internal sources.

In addition, a large part of the workforce must constantly be updated in


new techniques and developments. This task is the function of the
training and development sub-system.

Q&A-6.32
4.

5.

6.

Solved Scanner

CA Final Gr. II Paper - 6

Compensation :

This sub-system is concerned with the task of determining pay and other
benefits for the workers of the concern.

It makes use of traditional payroll and other financial records, government


reports and unions expectation before arriving at the final figures of pay
and other benefits for each category of workers.
Maintenance :

This sub-system is designed to ensure that personnel policies and


procedures are achieved.

It may be extended to the operation of systems of control work standards


which are required to measure performance against financial plans or
other programmes, and the many subsidiary records normally associated
with the collection, maintenance and dissemination of personnel data.
Health & Safety :

This sub-system is concerned with the health of personnel and the safety
of jobs in the organisation.

2001 - May [1] {C} (a) Discuss the various factors on which the information requirement
of executives depend.
(10 marks)
Answer :
System analysts develop different categories of information system to meet the various
business need
1. Transaction Processing System (TPS)
2.

Management Information System (MIS)

3.

Decision Information System (DSS)

4.

Executive Information System (EIS)

5. Expert System (ES)


Brief discussion of these systems are :1.

Transaction Processing Systems :

Transaction Processing systems are aimed at expediting and improving the


routine business activities that all organizations engage.

Standard operating procedures, which facilitate handling of transactions are


often embedded in computer programs that control the entry of data,
processing of details and presentation of data and information.

[Chapter # 1] Inform ation System Concepts O

2.

3.

4.

Q&A-6.33

The high volume of well-understood transactions associated with the operative


level of an organisation as well as the ability of the managers to develop
specific procedures for handling them often trigger the need for computer
assistance.

Transaction processing systems, if computerized, provide speed and accuracy


and can be programmed to follow routines without any variance.
Management Information System :

Management Information Systems assist managers in decision-making and


problem solving.

MIS use results produced by the transaction processing systems, but they
may also use other information. In any organization, decisions must be made
on many issues that recur regularly and require a certain amount of
information.

The manager can identify the information that will be needed for decisionmaking. In turn, the information systems can be developed so that reports can
be prepared regularly to support these recurring decisions.
Decision Support Systems :

DSS aim at assisting managers who are faced with unique(non-recurring)


decision problems.

They are of much more use when the decisions are of an unstructured or
semi-structured nature. In this situation, the problem area is modeled and
various alternatives are explored. In its simplest form, the spreadsheet could
be considered as a decision-support system.

Models can also be built using formulae and variables can be changed to see
what would be the outcome. Purpose-built decision support systems usually
incorporate more sophisticated modeling using statistical techniques such as
linear programming, regression analysis, time series analysis etc.

These decision-support systems are an integrated piece of software


incorporating database, model base, and user interface. W hile the decisionsupport system can be of use at the tactical level, it is the strategic level that
could make best use of it.
Executive Information Systems :

EIS are designed primarily for the strategic level of management. They enable
executives to extract summary data from the database and model complex
problems without the need to learn complex query languages, enter formulae,
use complex statistics, or have high computing skills.

These systems are easy to use, incorporating touch screens in some


instances, and being graphic based. High level summary data and trend
analysis is provided at the touch of a button, using graphics as a way of
presenting the information.

Q&A-6.34

Solved Scanner

CA Final Gr. II Paper - 6

5.

There are standard templates for doing these, the executives need not
construct the query or model.
Expert Systems :

Expert systems are designed to replace the need for a human expert. They
are particularly important where expertise is scarce and therefore expensive.

This is a software that expresses knowledge in terms of facts and rules.

This knowledge will be in a specific area, and therefore expert systems are not
general as are most decision support systems that can be applied to most
scenarios.

Expert systems have arisen largely from academic research into artificial
intelligence.

The expert systems are developed using very different programming


languages such as PROLOG. These systems are of greater use in the tactical
and strategic level.

2001 - May [2] (b) What is an Executive Information System (EIS)? Discuss its various
characteristics.
(5 marks)
Answer :
Executive Information System :

EIS is a tool that is designed to meet the special needs of top-level managers.

EIS provides direct on-line access to relevant information in a useful and navigable
format. Relevant information is timely, accurate, and actionable about aspects of
a business that are of particular interest to the senior manager.

The useful and navigable format of the system means that it is specifically
designed to be used by individuals with limited time, limited keyboarding skills, and
little direct experience with computers.

An EIS is easy to navigate so that managers can identify broad strategic issues,
and then explore the information to find the root causes of those issues.

EIS is mainly an advancement of MIS but it can include the Decision Support
System (DSS).

[Chapter # 1] Inform ation System Concepts O

Q&A-6.35

Characteristics of EIS :
1. An Executive Information System (EIS) is a tool that provides direct on-line access
to relevant information in a useful and navigable format.
2. EIS is a computer based information system that serves the information need of
top executives.
3. EIS enables users to extract summary data and model complex problems without
the need to learn query languages, statistical formulas or high computing skills.
4. EIS is very user friendly, supported by graphics and exception reporting and drill
down capabilities.
5. EIS provides rapid access to timely information and direct access to management
reports.
6. EIS is capable of accessing both internal data and external data.
7. EIS is easily connected to Internet (with online information services and e-mail)
8. EIS provides extensive online analysis tool like trend analysis, market conditions
etc.
9. EIS can easily be given a DSS support for decision making.
EIS is easy to navigate so that managers can identify broad strategic issues, and
then explore the information to find the root causes of those issues.
2001 - Nov [1] {C} (a) What are different categories of Information Systems? Explain
each one of them briefly.
(10 marks)
Answer :
Please refer 2001 - May [2] (b) on page no. 34
2002 - May [1] {C} (a) Discuss various constraints that come in the way of operating an
effective M.I.S. How these constraints could be avoided?
(10 marks)
Answer :
Major constraints which come in the way of operating an information system are the
following. The measure to overcome them are also mentioned.
Constraint
1.

Remedy

Lack of awareness : Middle level Management staff should be properly


managers may not be aware of the educated about the advantages of using
benefits and advantages of using the the MIS.
MIS. They are continue to complain
about the luck of information rather
then using the MIS

Q&A-6.36

Solved Scanner

CA Final Gr. II Paper - 6

2.

Non-availability of experts : Nonavailability of experts, who can


diagnose the objectives of the
organisation and provide a desired
direction for installing and operating
system.

This problem may be overcome by


grooming internal staff. The grooming of
staff should be preceded by proper
selection and training.

3.

Selection of Sub-system : Experts


usually face the problem of selecting
the sub-system of MIS to be installed
and operated upon.

Install proper criteria, which should guide


the experts here, may be the need and
importance of a function for which MIS
can be installed first.

4.

High Turnover of Experts : Such


problems generally arises due to
factors like. Salary, possibilities of
promotion, future g rowth, Top
managements behaviour.
Non- standard approach : Due to
varied objec tives of business
concerns, the approach adopted by
experts for designing and implementing MIS is a non-standardised one.

It can be reduced by establishing better


working conditions and providing
appropriate benefit and paying at least at
par with other similar concerns.

6.

Non-cooperation of staff : Nonavailability of cooperation from staff


in fact is a crucial problem in the
operation of an effective MIS.

It should be handled tactfully. Educating


the staff may solve this problem. This task
should be carried out by organising
lectures, showing films and also explaining to them the utility of the system.
Besides this, som e persons should also
be involved in the development and
implementation of the system.

7.

Quantification of benefits: Difficulty


in quantifying the benefits of MIS, so
that it can be easily comparable with
cost this raises questions by
departmental managers about the
utility of MIS. They forget that MIS is
a tool, which is essential to fight out
competition and the state of uncertainty that surrounds business today.

Manager should be educated that MIS is


a tool, which is essential to fight out
c o m p e t i ti o n a n d t h e s t a t e o f
environmental.

5.

Though in this regard nothing can be done


at the initial stage but by and by
standardisation may be arrived at, for the
organisation in the same industry.

[Chapter # 1] Inform ation System Concepts O

Q&A-6.37

2002 - Nov [2] (a) Differentiate among Strategic, Tactical and Operational categories
of Information required for different levels of Managerial decision-making. (8 marks)
(b) In what ways does an Executive Information system differ from the Traditional
Information system?
(12 marks)
Answer :
(a) Different level of information required for different level of managerial decisionmaking are :
1. Strategic-level information :

Strategic-level information systems help senior management to tackle


and address strategic issues and long-term trends, both within the firm
and external environm ent.

Their principal concern is matching changes in the external environment


with existing organisational capability - What will be the cost-trends,
where will our firm fit in, what products should be made etc?

In other words, these systems are designed to provide top-management


with information that assists them in making long-range planning
decisions for the organizastion.
2. Tactical - level information :

Tactical -level information systems serve middle level managers and help
in taking decisions for a period of 2-3 years.

The managers are typically concerned with planning, controlling and use
summaries of transactions to aid their decision-making. In other words,
these systems provide middle-level managers with the information they
need to monitor and control operations and to allocate resources more
effectively.

In tactical systems, transactions data are summarized, aggregated, or


analysed.

Their purpose is not to support the execution of operational tasks but to


help the manager control these operations.
3. Operational-level information :

Operational-level information systems are typically transaction processing


systems and help in the operational level managers to keep track of
elementary activities and transactions of the organisations such as sales,
receipts, cash deposits, flow of materials etc.

Their purpose is to answer routine questions and to track flow of


transactions.

Thus, the primary concern of these systems is to collect, validate, and


record transactional data describing the acquisition or disbursement of
corporate resources.

Q&A-6.38

Solved Scanner

CA Final Gr. II Paper - 6

Thus, each type of information system serves the requirements of a


particular level in the organisation, providing the needed basis for decision
making.
DECISION TYPES

INFORMATION
NEEDS
External,
Summarized,
Wide Scope,
Unstructured,
Ad hoc.

Unstructured
(Strategic Planning
Decision)
Semi-structured
(Management
Control
Decision)

Internal
Detailed,
Narrow Focus,
Scheduled,
Routine

Structured
(Operational Control
Decision)

Above diagram comprehensively shows, various aspects of Information System and


Managements information needs.
Difference between Strategic, Tactical and Operational categories of information
required for different level of managerial decision making :
Levels of
Management

Strategic Level

Tactical Level

Operating Level

Decision
makers

To p M a n a g e m e n t / Middle Level
Board of Directors
management

Supervisors, foremen,
operating management

Responsibilities

Defining objectives,
Policies for whole
org,
lo n g - t e rm
planning, Large scale
investment decisions,
Appointment of middle
management

Effective use of
existing facilities and
r e s o u r c e s w i t h in
budget constraints and
prescribed objectives,
and Making routine
day-to-day decisions

Decision
Types

Un-Structured Using Decision Flow between S t r u c t u r e d U s i n g


Quantitative
a n d top management and Largely Quantitative
Qualitative Information low level management
Data

Planning Range

Extensive

Esta b l i s h m e n t and
monitoring of budgets,
Acquisition of resources,
developing operational
policies, and objectives,
Appointing staff

Planning Flow between Narrow


top management and
low level management

[Chapter # 1] Inform ation System Concepts O


Breadth
of Control

Extensive

Q&A-6.39

Control Flow between Narrow


top management and
low level management

Type of External Market and


Costs and sales
Information
economic
analyses,
Required
forecasts, political
performance
and social trends,
measures,
legislative,
Summaries of
technological
operations/Productio
constrains and
n, budget/ actual
opportunities.
comparisons etc.
Exceptional reports,
Ratio Analysis,
Cash/ Fund Flows
Project status
report.

Not much
information
required from
external source
Sales orders, prod
uction
requirements,
customer credit
status, deliveries,
dispatches etc.

Decision
Examples

Production scheduling
Maintenance,
Reordering Credit
approval.

Mergers and
acquisitions, New
product planning,
Capital investments,
Financial structuring.

Pricing, capacity
planning, Budget
preparation,
Purchasing contracts.

(b) An Executive Information System is a tool that provides direct on-line access to
relevant information in a timely manner, accurate, and actionable information about
aspects of a business that are of particular interest to the senior manager. Where
as Traditional Information System can be considered as a set of elements (people,
procedures and resources) that provides effective information for decision making
and/or control of some functionalities of an organisation.
Distinction between Executive Information Systems and traditional information
systems
Basis
Purpose

Executive Information
System

Tailored to meet executives General purpose systems


information needs

Assessment of Issue/problem specific


data
access and aggregation
Tool

Traditional systems

data Aggregation and data access is


open

On-line analysis tools including Basic query processing tools


trends analysis and exception
reporting

Q&A-6.40

Solved Scanner

CA Final Gr. II Paper - 6

Scope of data Access to both internal and Access to only internal data
analysis
external data
User Interface

User interface is either mouse User interface was keyboard driven


or touch-screen driven

Complexity

Ease of use by executives Executives need IT department


without assistance
assistance

Presentation

(i) Information presented by (i) Information presented as twopictorial or graphical means


dimension tables
(ii) Information is presented in (ii) Data aggregation on predetersummary format, e.g sales
mined level of detail, change
for the whole company.
requires programming skills
There is facility to drill
and design by a separate IT
down to other levels of
department.
information to see how the
sale figure were arrived at by geographical location, by
product group etc. (although
individual transactions are
not usually available). T the
ability to manipulate data, to
project what data, to project
what if outcomes and to
work with modeling tools
within the system are also
evident in EIS

2002 - Nov [4] (c) What systems interfaces are involved in Production Planning?
(4 marks)
Answer :
In order to understand the system interfaces in Production Planning, we study it in
various steps.

[Chapter # 1] Inform ation System Concepts O

Q&A-6.41

1. Production Planning :

Under Cost Estimation System, a customer order is received by a firm after


a customer has accepted an estimate of the cost of a job.

The cost estimation system provides with manufacturing cost estimates based
on enquiries received from potential customers.

Cost estimates are primary input when determining a price that will be quoted
to a customer.

W hen an order is received, the cost estimation system forwards the budgeted
costs to the production scheduling system.

Interfaces between the cost estimation system and other systems are shown
in figure. No functions within the cost estimation system result in general
ledger entries.

2.

System interfaces :

After the order is received, materials, machine time and labour requirements
for the job are then estimated and costed using standard costs.

Overhead costs are also estimated and the job cost estimate is then returned
to the system. This estimate serves as the basis for the selling price that are
quoted to the customer.

Q&A-6.42
3.

Solved Scanner

CA Final Gr. II Paper - 6

Files and inputs :

The cost estimation system requires a standard cost file, in which standard
costs are maintained for each labour, material and machine classification as
well as for standard manufacturing operations. These costs are used for
preparing manufacturing cost estimates.

The standard cost are updated based upon the information provided by the
budgeting system .

The file on bill of materials contains the description of all materials required to
produce each product. The file also contains a list of approved vendors.
4. Reports :

The cost estimation system can prepare manufacturing cost estim ation reports
for each job and performance measurement report which shows the various
cost estimation reports used in processing it and the number of orders
received.

The estimated cost and actual cost are compared for each cost classification.
This enables over or underestimation to be identified.
2003 - May [2] (a) Discuss the effect of applying computer technology to Management
Information system.
(10 marks)
(b) Explain the concept of decomposition with the help of an example. (10 marks)
Answer :
(a) Effects/ Benefits of using computer for management information system (MIS) can
be describe as follows :
1. Provides More comprehensive information : The use of computer for MIS,
enabled systems expert to provide more comprehensive information to
executives on business matters.
2. Increases the effectiveness of information system :

Information received in time is of im mense value and importance to a


concern.

Prior to the use of computer technology for information purposes, it was


difficult to provide the relevant information to business executives in time
even after incurring huge expenses.

The use of computer technology has overcome this problem. Now, it is


not difficult to provide timely, accurate and desired information for the
purpose of decision making.
3. Speed of processing and retrieval of data increases :

In present scenario business are characterised by high degree of


complexity, keen competition and high risk and reward factors.

This invariable calls for systems capable of providing relevant information


with minimum loss of time.

[Chapter # 1] Inform ation System Concepts O

4.

5.

6.

Q&A-6.43

Manual systems, howsoever well organised, often fail to match the


demand for information for decision making.

Com puters with their unbelievably fast computational capability and


systematic storage of information with random access facility have
emerged as an answer to the problem s faced in modern days
management.
Increases the Scope of analysis :

The use of computer can provide multiple type of information accurately


and in no time to decision makers. Such information equips an executive
to carry out a thorough analysis of the problems and to arrive at the final
decision.

Computer is capable of providing various types of sales reports for


example; area wise sales commission of each salesman, product-wise
sales, etc.

These reports are quite useful in analysing the sales department working
and to ascertain their weaknesses so that adequate measures may be
taken in time.

In this way, use of computer has increases the scope of analysis.


Scope of use information system has expanded :

The importance and utility of information system in business


organisations was realised by most of the concerns, specially after the
induction of computers for MIS development.

System experts in business organisations developed areas and functions


where computerised MIS could be used to improve the working of the
concern.

This type of applications hitherto are not feasible under the manual
system.
Complexity of system design and operation increased :

The need of highly processed an sophisticated information based on


multitudes of variables has made the designing of the system quite
complex. During the initial years, after the induction of computer for MIS
development, systems experts faced problems in designing systems and
their operations.

The reason at that time was the non-availability of experts required for the
purpose. But these days the situation is better. The computer
manufacturers have developed some important software to help their
users.

Some private agencies are also there who can perform the task of
developing programs, to cater to the specialised needs of their
customers, either on consultancy basis or on contract.

Q&A-6.44
7.

Solved Scanner

CA Final Gr. II Paper - 6

Integrates the working of different information sub-systems :

A suitable structure of management inform ation system m ay be a


federation of information sub-system, viz. production, material, marketing,
finance, engineering and personnel.

Each of these sub-systems are required to provide information to support


operational control, management control and strategic planning.

Such information may be made available from a common-data-base. This


common data base may meet out the information requirements of
different information sub-system by utillising the services of computers for
storing, processing, analysing and providing such information as and
when required.

In this way computer technology is useful for integrating the day-to-day


working of different information sub system.
(b) Decomposition :

A complex system is difficult to comprehend when considered as a whole.


Therefore the system is decomposed or factored into subsystem s.

The boundaries and interfaces are defined, so that the sum of the subsystems
constitutes the entire system.

This process of decomposition is continued with subsystems divided into


smaller subsystems until the smallest subsystems are of managable size.

The subsystems resulting from this process generally form hierarchical


structures. In the hierarchy, a subsystem is one element of supra-system (the
system above it)

[Chapter # 1] Inform ation System Concepts O

Q&A-6.45

An example of decomposition is the factoring of an information processing


system into subsystems. One approach to decomposition might proceed as
follows :
1. Information system divided into subsystem such as :
(a) Sales and order entry.
(b) Inventory.
(c) Planning.
(d) Purchasing.
(e) Production.
(f) Personnel and payroll.
(g) Accounting and control.
(h) Environmental intelligence.
2. Each subsystem is divided further into subsystems. e.g. the personnel and payroll
subsystem might be divided into the following smaller subsystems:
(a) Creation validation and updation of personnel pay-roll records.
(b) Personnel reports.
(c) Personnel on Payroll data entry and validation.
(d) Hourly payroll processing.
(e) Incentive Salaried payroll processing.
(f) Payroll reports for management.
(g) Payroll reports for concerned government departments.
3.
If the task is to design and program a new system, the subsystems (major
applications) defined as above, might be further subdivided into smaller
subsystems or modules.

The hourly payroll processing subsystem might be factored into modules for
the calculation of deductions and net pay, payroll register and audit controls
preparation, cheque printing, and register and controls output.

Decomposition into subsystems is used to analyse an existing system and to


design and implement a new system.

Q&A-6.46

Solved Scanner

CA Final Gr. II Paper - 6

In both cases, the investigator or designer must decide how to factor, i.e.,
where to draw the boundaries. The decisions will depend on the objectives of
the decomposition and also on individual differences among designers: the
latter should be minimized.
The general principle in decomposition, which assumes that system objectives
dictate the process, is functional cohesion. Components are considered to be
part of the same subsystem if they perform or are related to the same function.
As an example, an application program to be divided into modules
(subsystems) will divide along major program functions such as accumulating
hours worked, calculating deductions, printing a cheque, etc. in design, the
identification of functionally cohesive subsystems is the first step. The
boundary then needs to be clearly specified, interfaces simplified, and
appropriate connections established among the subsystems.

2003 - May [3] (b) Discuss various benefits which are attained by implementing a
computerised model for making decision.
(10 marks)
Answer :

Effective decision making process in the global scenario can not be possible
without effective use of Management Information System (MIS).

A computerized model may incorporate accounting, production, transportation,


manufacturing and marketing operations of the company. Such model allow a
decision maker to consider a large number of factors in decision-making process,
which was not possible in manual system.

This highlights the need to reduce skepticism that managers have toward
sophisticated computerized decision making aids.

This type of model puts pertinent information into a analytical framework that aids
the management decision making process.
Some of the benefits offered by computerized model for decision-making are:
Benefits
1.

Providing
Prompt
Information.

Explanations
Manager must deal immediately with many day-to- day
business problems, as well as plan and control their
operations. Managers require different information for the
various kinds of decisions they must make. Computerised
model provides reports on variances, cost-volume-profit
analysis, etc to help managers in decision-making.

[Chapter # 1] Inform ation System Concepts O

Q&A-6.47

2.

Helps in Data Computerised models provide managers with data to which


ratios and analysis tools may be applied. They provide
Analysis
management with a variety of measures of the soundness of
the organization and make it possible to explore ways of
improving the organizations financial condition.

3.

Helps in
casting

4.

Helps in
variance
analysis

Compterised models permit managers in tracing variances ie.


actual revenues and expenses and compare these amounts to
expected revenues and expenses and compare these amounts
to expected revenues and expenses. Comparison of budget
data against such standards allow managers to assess how
they use their resources to achieve their goals.

5.

Facilitates
cashmanagement

The models also estimate the amount of cash that will be


received and spent each month i.e. in which months there will
be excess funds that might be put to use and in which month
there will be insufficient funds. Thus, the information supplied
helps managers to make decisions about investment,
purchasing and borrowing money. The models also help in
tracking cash balances on a day-to-day basis and help the
managers in their investment decisions so as to maximize
organization income.

6.

Providing data Computerized models are highly useful for carrying out
b a s e & it s analysis of historical data speedily, which may be quite difficult
or even impossible manually for forecasting the future.
Interface

For Computerised models make a fairly accurate forecasts that will


affect the organization performance in the future. Forecasting
the financial health of the organization through long-range
budget estimates, provides managers with opportunities to
consider actions that will help the organization survive bad
times or take advantage of a future environment.

2003 - Nov [2] (a) Explain the role played by Financial Information System in making
financial decisions.
(8 marks)
(b) (i) Describe briefly three levels of Management.
(3 marks)
(ii) Mention atleast two pieces of information one internal and one external
required at every one of the levels of Managem ent.
(6 marks)
(iii) Discuss the potential impact of computers and MIS at the top level of
Management.
(3 marks)

Q&A-6.48

Solved Scanner

CA Final Gr. II Paper - 6

Answer :
(a) Firstly, we try to understand the meaning and scope of Financial Decision Making.

Role of Financial Information System in making Financial Decision


Roles
1.

Explanations

Estimation of requ-
irements of funds :

This is the very important and starting point of making


financial decisions.
A very careful estimation of funds and the time at
which these funds are required is made in this stage.
This can be done by forecasting all physical activities
of the firm and translating them into monetary units.

2.

Capital budgeting
decisions:

Funds procured from various sources are required to


be invested in different assets.
W ith the help of capital budgeting, decision maker
can determine feasibility of investment in long-term
assets. This will help in attainment of financial
objectives.

3.

Capital structure
decisions :

Decisions are to be taken to select an optimum m ix


of different sources of capital structure. There are
many options available for procuring funds.

[Chapter # 1] Inform ation System Concepts O

Q&A-6.49

Decision maker has to decide the ratio between debt


and equity, long-term and short-term funds etc.
He has to ensure that overall capital structure is such
that the company is able to procure funds at optimum
cost.

4.

Wo r k in g
cap ital
management :

Working capital management is concerned with the


investment of long term funds into current assets.
Decisions are to be taken for effective financing of
current assets required for day-to-day running of the
organization.

5.

Current assets ma-


nagement :

Policy decisions are taken regarding various items of


current assets. Credit policy determines the amount
of sundry debtors at any point of time.
Inventory policy is to be determined jointly between
finance and production department.

6.

Tax management :

7.

Profit planning :

Tax planning is aimed at reducing of outflow of cash


resources by way of taxes so that the same may be
effectively utilized for the benefit of business.
The purpose of tax planning is to take full advantage
of exemptions, deductions, concessions, rebates,
allowances and other relief.
This part of profit planning is essential for the growth
of the organization.
The decisions maker has to make decision regarding
profits and dividends.
He has to ensure adequate surplus in future for
growth and distribution of dividends.

(b)
(i) Three levels of management are briefly discuss below :
Level & Management
1.

Top level
Management
(Strategic level)

Explanation

Strategic level is defined as set of management


positions that is concerned with developing of
organisational missions, objectives and strategies,
directing and managing the organization in an integrated
manner.

Q&A-6.50

Solved Scanner

2.

Middle Level
Management
(Tactical Level)

3.

Supervisory
level
Management
(Ope-rating
Level)

CA Final Gr. II Paper - 6

Decisions made at this level of organization to handle


problems critical to the survival and success of the
organization are called strategic decisions.
Strategic level also establishes a budget framework
under which the various departments will operate.
Tactical level lies in the middle of managerial hierarchy.
At this level, managers plan, organize, lead and control
the activities of other managers.
Decisions made at this level, called the tactical
decisions, are made to implement strategic decisions.
Tactical decisions are relatively short, step-like spot
solutions to breakdown strategic decisions into
implementable packages.
Operating Level is the lowest level in managerial
hierarchy. The managers at this level coordinate the
work of others who are not them selves managers.
At supervisory level, managers are responsible for
routine, day-to-day decisions and activities of the
organisation which do not require much judgement
discretion.
They ensure that specific tasks are carried out
effectively and efficiently.

Characteristic of different Levels of management :


Sl.
No.

Characteristic

Top Management

Middle
Management

Operating
Management

Focus of
Planning

Very heavy

Heavy

Moderate
Minimum

Focus on
Control

Moderate

Heavy

Heavy

Scope of
activity

Extremely broad

Entire functional Single subfuncarea


tion or sub-task

Time frame

One to five years

Upto one year

Day-to-day

[Chapter # 1] Inform ation System Concepts O

Q&A-6.51

Type of
information

Mostly external

I n te r n a l , m o r e Internal historical,
accurate
high level of
accuracy

Nature of
activity

Relatively
unstructured

Moderately
Structured

Level of
complexity

V e r y c o m p l e x Less comple x Straight forward


many variables
better defined

Job
measurement

Difficult

Less difficult

Relatively easy

Mental
attributes
effective

Creative,
administrative

Responsive
innovative

Efficient
Persuasive

10

Number of
people involved

Few

Moderate
number

Many

11

Result of
activity

Plan, Policies and Implementation


Strategies
schedules,
performance,
yardsticks

12

Department/
divisional
inter action

Intra-division

Highly structured

End product

Intra-department Inter- department

(ii) Management at different levels takes decision matching to their position or


hierarchy in the organisation. And different types of information systems are
designed and developed for them i.e. an information system for a marketing
manager will have different characteristics then information system for a managing
director.
Types of Information
Generally information requirement at various levels of management can be divided
into two broad categories.
1. External Information : This information is obtained from outside the organization
boundary, or we can say this information is related with the environment of
organization, in which organization operate. The environment information primarily
includes the followings:
Environment Information :

Government Policies: Information about concessions, benefits, restrictions


of government policies in respect of tax concessions or any other aspects,
which may be useful to the organization in the future period.

Q&A-6.52

Solved Scanner

CA Final Gr. II Paper - 6

2.

Major factors of production: Information related with source, cost, location,


availability, accessibility and productivity of the m ajor factors of production viz.
(i) labour (ii) materials and parts, and (iii) capital.

Technological environment : Forecast of any technological changes in the


industry and the probable effect of it on the firm.

Economic Trends : It includes information relating to economic indicates like


consumer disposal income, environm ent, productivity, capital investment etc.
such information is valuable for those firms, especially, where output is a
function of these important variables.
Internal Information : This information is part of internal functioning of
organization, and this generated form various internal functional areas of
organization; like Marketing, Production, and Finance, etc.

(iii) Please refer 2003 - May [2] (a) on page no. 42

[Chapter # 1] Inform ation System Concepts O

Q&A-6.53

2004 - May [2] (c) Explain three broad categories of the planning information
requirements of executives.
(10 marks)
Answer :
Please refer 2003 - Nov [2] (b) (ii) on page no. 47
2004 - May [5] (a) "The Personnel information system is the backbone of any
organisation." Explain.
(2 marks)
(b) Discuss various sub-systems of PIS, which are responsible to increase its
operational efficiency.
(6 marks)
Answer :
(a) Please refer 2000 - Nov [1] {C} (a) on page no. 30
(b)
The personnel information system deals with the flow of information about
people working in the organisation as well as future personnel needs.

In most of the organisations, the system is concerned primarily with the six
basic sub-systems of the personnel function.
1. Recruitment.
2. Placement.
3. Training.
4. Compensation.
5. Maintenance and
6. Health and safety.

It is generally accepted that the personnel function is one of the least


computerised of all the functions. Automated system may not be necessary
for small, but large business firms are realising that computer based personnel
information systems are necessary for increasing the operational efficiency of
personnel managem ent.

The personnel system should be organised on functional basis. It should have


the following information sub-systems to increase the operational efficiency
of personnel managem ent.
1.

Recruitment

2.

Placement

Properly managed recruitment sub-system may forecast


personnel needs and skills required for recruiting
personnel at the proper time to meet organisational
manpower needs.
Such a sub-system may not only furnish information
concerning skills required for company programmes and
processes but also maintains the inventory of skills
available within the organisation
This sub-system is concerned with the task of matching
the available persons with the requirements.

Q&A-6.54

Solved Scanner

3.

Training
and
development

4.

Compensation

5.

Maintenance

6.

Health & Safety

CA Final Gr. II Paper - 6

A good placement sub-system makes use of latest


behavioral tools and techniques.
It ensures that the capabilities of people are identified
before being matched with properly organised work
requirements.
As technological changes and demands for new skills
accelerate, many componies find that they must develop
much of their requirements from internal sources.
In addition, a large part of the workforce must constantly
be updated in new techniques and developments.
This task is the function of the training and development
sub-system.
This sub-system is concerned with the task of
determining pay and other benefits for the workers of the
concern.
It makes use of traditional payroll and other financial
records, government reports and unions expectation
before arriving at the final figures of pay and other
benefits for each category of workers.
This sub-system is designed to ensure that personnel
policies and procedures are achieved.
It may be extended to the operation of systems of control
work standards which are required to measure
performance against financial plans or other
programmes, and the many subsidiary records normally
associated with the collection, maintenance and
dissemination of personnel data.
This sub-system is concerned with the health of
personnel and the safety of jobs in the organisation.

2004 - Nov [2] (a) W hat are the production information requirements of a GM
(Production and Operations Management) with regard to production planning and
control?
(10 marks)
(b) W hat are the variables that the top management should consider during
negotiation with the labour unions?
(5 marks)
(c) Successful executives take decisions relying more on intuition than on any
quantitative analytical decision technique. Mention five characteristics of the types
of information that are responsible for this phenomenon in executive decisionmaking.
(5 marks)

[Chapter # 1] Inform ation System Concepts O

Q&A-6.55

Answer :
(a) The production planning and control system involves two main procedures.
1. The specification of materials and operations requirements and production
scheduling. The information requirements of a GM for the materials and
operations phase of any organisation are as stated below:
(i) Firms policy with regard to production of various products.
(ii) Sales order, sales forecast, stock position, order backlog.
(iii) Available labour force with their capabilities.
(iv) Standards of labour time, material, machine time and overhead cost etc.
(v) Schedule of meeting the sales orders, region-wise, territory-wise etc.
(vi) Quality norms for materials to be used and for the finished products.
(vii) Break-up of the jobs and their resource requirements.
2. Planning the specific time at which product items should be m anufactured is
known as production scheduling. The information is required by the
production-scheduling department so as to meet the following objectives :
(i) To determine the stages of production in sequential and rational order ;
(ii) To minimise the idle time on the part of the operators and equipments;
(iii) To assess the extent of need for subcontracting to outside parties;
(iv) To ensure that completion dates and target dates of completing the
production plan are met fully; and
(v) To study alternative methods to performing the activities so that time
taken to perform can be further reduced.
(b) In big or meddle level of organisation, generally labours are unionized.
Unionized organisations usually have strict regulations regarding such items as pay
scales, hiring and firing, promotions and working conditions. Management has
choice of trade - offs on the following variables during negotiations with the labour
unions
(i) Wage raise
(ii) Paid holidays
(iii) Contribution to employees, insurance and pension plan
(iv) Overtime premiums.
Cost accountants/payroll accountants would be in the best position to make
various estimates for the cost implication of trade off.
(c)

Top executives are generally rely much more on their own intuition, gut
feelings and past experience rather than on sophisticated analytical skills.
The type of decisions that the executives must make is broad. Often,
executives make these decisions based on a vision they have regarding what
it will take to make their companies successful.

Q&A-6.56

Solved Scanner

CA Final Gr. II Paper - 6

The intuitive character of executive decision-making is reflected strongly in the


types of information found most useful to executives.
Five characteristics of the types of information used in executive decisionmaking are as follows:-

Characteristics

Used in executive decisions

1.

Low level of detail

Most important executive decisions are made by


observing broad trends. This requires the executive to be
more aware of the large overviews than the tiny items.
Even so, many executives insist that the answers to some
questions can only be found by mucking through details.

High degree
uncertainty

Lack of structure

Many of the decisions m ade by executives are relatively


unstructured. For instance, what general direction should
the company take? Or what type of advertising campaign
will best promote the new product line? These types of
decisions are not so clear-cut as deciding how to debut a
computer program or how to deal with an overdue
account balance. Also, it is not always obvious which data
are required or how to weigh available data when
reaching at a decision.

Future orientation

Strategic-planning decisions are made in order to shape


future events. As conditions change, organisations must
also change. It is the executives responsibility to make
sure that the organisation keeps pointed toward the
future. Some key questions about the future include:
How will future technologies affect what the company is
currently doing? W here will the economy move next, and
how might that affect consumer buying patterns? As one
can see, the answers to all of these questions about the
future external environment are vital.

Informal source

Executives rely heavily on informal source for key


information. For exam ple. Lunch with a colleague in
another firm might reveal some important competitor
strategies. Some other important information sources of
information are meetings, tours around the companys

o f Executives work in a decision space that is often


characterized by lack of precedent.

[Chapter # 1] Inform ation System Concepts O

Q&A-6.57

facilities to chat with employees, brainstorming with a


trusted colleague or two, and social events. Informal
sources such as television might also feature news of
momentous concerns to the executives.
2005 - May [2] (b) "A decision support system supports the human decision-making
process rather than providing a means to replace it". Justify the above statement by
stating the characteristics of decision support system.
(5 marks)
(c) Describe the main pre-requisites of a Managem ent Information System which
makes it an effective tool.
(5 marks)
Answer :
(b)

Decision Support System are an application of Herbert Simon Model. The


model has three phases, viz. Intelligence, Design and Choice.
The decision support system basically helps the information system in the
intelligence phase where the objective is to identify the problem and then go
to the design phase for solution. The choice of selection criterion varies from
problem to problem.
It is, therefore, required to go through these phases again and again till a
satisfactory solution is found. In the use of three phase cycle, you may use
either inquiry, analysis, and models or accounting system to come to a rational
solution.
The decision support system helps in making a decision and also in its
performance evaluation. These systems can be used to validate the decision
by performing sensitivity analysis on various parameters of the problem.
The decision support system refers to a class of system which support in the
process of decision making and does not always give a decision itself. The
nature of the decision is such that the decision m akers need a variety of
information, when same or similar types of decisions are to be made.
These needs are such that wider additional demands on information would be
made, the moment a piece of information is received. The calls on the
information are continuously made till the decision maker is fully satisfied.
The reason for changing the demands is also because the methods of
decision making undergo a change from time to time.
The quantum and the scope of information also changes depending upon the
risk in decision making. The higher the risk, more information may be sought.
A decision support system (DSS) can be defined as a system that provides
tools to managers to assist them in solving semi structured and unstructured
problems in their own, somewhat personalised, way.

Q&A-6.58

Solved Scanner

CA Final Gr. II Paper - 6

Often, some type of modeling environment perhaps a very simple environment


such as the one accompanying a spreadsheet package- is involved. A DSS
is not intended to make decisions for managers, but rather to provide
managers with a set of capabilities that enables them to generate the
information required by them in making decisions.

In other words, a DSS supports the-human decision-making process rather


than providing a m eans to replace it.

Technically, a DSS does not need to involve high technology.

Systems that replace hum an decision making - rather than support it - are
sometimes called programmed decision systems. These systems are used to
make routing, structured decisions, such as approving loans or credit,
reordering inventory, triggering reminder notices, and selecting audit samples.
In programmed decision systems, the focus is on doing something more
efficiently. On the other hand in decision support systems, the focus is on
helping decision makers become more effective.
Characteristics of DSS :Please refer 2002 - Nov [7] (c) on page no. 16
(c) Please refer 1999 - Nov [1] {C} (a) on page no. 29
2005 - May [4] (a) A Company is planning to introduce a new range of products. The
top management is advised to get developed on marketing information system which
can enhance the decisional capacities in various marketing activities. You being
in-charge of this project suggest what information sale-systems are required to be
developed.
(10 marks)
Answer :
Marketing Information System :

It is aimed at supporting the decision making, reporting and transaction processing


requirements of marketing and sales managem ent.

It consists of following inter-related information sub-systems to enhance the


decision-making capacities in various marketing activities:
1.

Sales

The objective of the sales manager is to coordinate the sales


effort so that the long-term profitability of the company is
maximized.
Decisions are required in the area of adequate stocks,
effective distribution channels, effective motivation of sales
personnel, promotion of more profitable products or product
lines and good customer relations.
Information required for analysis and support of sales is as
stated below:

[Chapter # 1] Inform ation System Concepts O

Q&A-6.59

(a) Sales Support - Sales support information system provides


information to sales personnel about the following:
(i) Product descriptions and performance specifications
(ii) Product prices;
(iii) Quantity discounts
(iv) Sales incentives for salespersons
(v) Sales promotions;
(vi) Strengths and weaknesses of competitors product;
(vii) Products inventory levels
(viii) The histories of customers relations with the company.
(ix) Sales policies and procedures established by the
company
(x) Buying habits of customers
(b) Sales Analysis -The sales analysis is a major activity in most
companies involved in sales. Its purpose is to provide
information for analysis of
(i) Product sales trends;
(ii) Product profitability on the product-by-product basis;
(iii) The performance of each sales region and sales
branch;
(iv) The performance of respective sales persons.
Information for sales analysis is derived primarily from
the sales order entry system; the majority of information
from actual sales transactions and is contained on
sales invoice. It includes information on product type,
product quality, price, customer identity and type, sales
region and salesperson etc.
The objective of marketing research is to investigate problems
2. Market
research and confronting the other managers in the marketing function. These
problems may involve sales, product development, advertising
intelligence
and promotion, customer service or general marketing
managem ent needs.
To satisfy these decision-making and reporting requirements,
the market research department must either periodically or
upon demand gather information from a wide variety of
sources.
The investigations undertaken by market research helps in
satisfaction of following informational needs of managers.

Q&A-6.60

Solved Scanner

CA Final Gr. II Paper - 6

(i) Information about the economy and economic trends


and the probable impact of these trends on emand for
the product.
(ii) Information about the past sales ; and sales trends for
the entire industry.
(iii) Information about potential new markets for product.
(iv) Information about com petitors, its product, strength,
weaknesses, new product plans, strategies and so on.
3. A d v e r t i s i n g The decision-maker can use the information provided by
marketing research in a number of ways for decision making
and
process.
Promotions
The promotion and advertising development devotes its
attention to planning and executing advertising campaign and to
carry out various product promotions. This includes:
(i) Promotion through limited budget.
(ii) Allocating resources in most effective manner.
(iii) Analysing an array of information, sales people, locations,
products, styles, sizes etc.
(iv) Storing information that can be combined with past
experiences of managers.
(v) Establishing a body of knowledge on the respons of market
for each of the several types of promotional activities such
as coupons, contests, trade show;
(vi) Continuously refreshing and modifying the information
base in accordance with rapid changing environm ent.
4.

Product
Development and
Planning

Product development involves analysing a possible


opportunity for a new product and evaluation preferred
specifications and probable market success.
Often the market research activity initially perceives the
opportunity and passes along information about it to the new
product development group. Alternatively.
(i) Sales persons may be aware of their need for a new
product;
(ii) Customer call reports may help elicit information about
new product needed which may encourage sales
persons to think about new product possibilities.
(iii) Sales analysis system indicates the most desirable
characteristics for the new product.

[Chapter # 1] Inform ation System Concepts O

Q&A-6.61

(iv) Market researchers gather information about the size


and structure of the market place for the product.
The product development department uses all these
information to develop specifications for a new product. Product
planning system provides marketing managem ent with packaging,
promotion, pricing and style recommendations throughout the life
of product.
5.

Product
Pricing
System

6.

Customer
Service

Product pricing is a complex managerial activity that is


affected by product cost, customer demand, market
psychology, competitors, prices and various actions taken by
competitors.
Prices may be determined on a full cost or marginal cost
basis which is usually seen as the starting point in setting
prices.
Pricing information system almost always utilizes information
about product cost.
Past sales profitability information is useful to help in
determining how much prices should be adjusted for changes
in cost to ensure that margins are maintained.
The objective of marketing department is to satisfy customers
with product and customer service.
To achieve these objectives, management provides
custom ers with technical assistance and product
maintenance.
Decisions are required in the area of training of service
personnel, capabilities of equipment and location
facilities to serve customers and assist in the dissemination
of technical information to the customers.
These decisions must be congruent with the marketing
management strategy regarding customer satisfaction and
service.

2005 - Nov [2] (a) How system approach can be used for solving problems?
(10 marks)
(b) Define the term system stress and system change.
(5 marks)
Answer :
(a)
The system approach to management is in fact a way of thinking about
management problems. It visualizes an organization as a group of interacting
and interdependent parts with a purpose.

Q&A-6.62

1.

2.

3.

Solved Scanner

CA Final Gr. II Paper - 6

Managers are not in a position to deal with individual parts separately since
action of one part is going to affect other parts.
Each problem should be examined in its entirety to the extent possible and
econom ically feasible from the point of view of the overall system of which the
problem under consideration is one part.
Under this approach, a manager should make conscious attempt to
understand the relationship among various parts of the organization and their
role in supporting the overall performance of the organization.
Before solving problem in any financial area, or in any specific sector of the
organization he should understand fully how the overall system would respond
to changes in its components parts.
To solve problems managers must view the organisation as a dynamic whole
and they should anticipate the intended as well as unintended impacts of their
decisions. By using systems approach management will understand that they
do not solve individual problems. Rather, they intervene in a system of
interrelated parts and the managerial functions.
In order to find out a solution for the problem by applying systems approach,
we should make use of the following six steps.

Defining of the problem: The problem involved here is of inordinate delay


between the receipt of orders and their delivery. This problem affects the vendor
in many ways, e.g., a bad reputation, loss of customers, reduction in profitability
and even stoppage of due payments.
Gathering and analyzing data concerning the problem : The problem of delay
in meeting orders, in this case, say, arises due to the following reasons :
(a) Excessive orders in the hand of vendors.
(b) Shortage of power (fall in production due to shortage of power).
Identification of alternative solutions : To over-come the stated problem by
system approach, suppose the following two solutions exist.
(a) Refusal of orders, in case the total size of the orders exceeds the plantcapacity one shift.
(b) To run the plant in double shift, to meet the commitment in time. Any shortfall
in power supply may be met be installing a generator.

[Chapter # 1] Inform ation System Concepts O


4.

Q&A-6.63

Evaluation of alternative solution : Out of the two identified solutions under the
preceding step, the second solution say accounts for an overall increase in the
profitability of the concern after off- setting additional cost for the generator produce
power. It also helps in retaining customers and growth of the concern.
5. Selection of the best alternative : Under this step, management more closely
examines the alternatives and puts its stamp on the best possible alternative. In
this case say the second alternative is finally chosen.
6. Implementation of solution : The implementation of the solution requires the
necessary policy changes. Besides this, the resources required to run the plant in
double shift and installation of a generator also are to be arranged. Finally,
appropriate procedures are developed to exercise smooth production and timely
supply to customers, the concerned officers are accordingly instructed.
(c) System stress and system change

Systems whether they be living or artificial systems, organisational systems,


information systems or systems of controls, change because they undergo
stress.

A stress is a force transmitted by a systems supra-system that causes a


system to change, so that the supra-system can better achieve its goals.

In trying to accommodate the stress, the system may impose stress on its
subsystems and so on.
Types of Stress : There are two type forms of stresses which can be imposed on a
system separately or concurrently.
1. A change in the goal set of the system. New goals may be created or old goals
may be eliminated,
2. A change in the achievement levels desired for existing goals. The level of desired
achievement may be increased or decreased.
e.g., the goal set for a computer system may change if a requirement is imposed by
management (the supra-system) for system data to be shared among m ultiple users
rather than be available only to a single user.
Consequences of Stress : W hen a supra-system exerts stress on a system, the
system will change to accomm odate the stress, or it will become pathological; that is,
it will decay and terminate.
Process of Adaptation : System s accommodate stress through a charge in form; there
can be structural changes or process changes. e.g., a computer system under stress
for more shareability of data may be changed by the installation of terminals in remote
locations (a structural change). Demands for greater efficiency may be met by changing
the way in which it sorts data (a process change).
It is very unlikely that system changes to accommodate stress will be global change to
its structure and processes. Instead, those responsible for the change will attempt to
localize it by confining the adjustment processes to only one or some of its subsystems.

Q&A-6.64

Solved Scanner

CA Final Gr. II Paper - 6

2005 - Nov [3] (a) What is an Executive Information system? Discuss its various
purposes.
(10 makes)
Answer :
Executive Information System :
1. Executive Information System is a DSS specifically designed to meet the special
needs of top-level Managers. EIS is a tool that provides direct on-line access to
relevant information in a useful and navigable format.
2. Relevant Information refers to timely, accurate and actionable information about
aspects of a business that are of particular interest to the executives.
3. The useful and navigable format of the system means that it is specifically
designed to be used by individuals with limited time, limited keyboarding skills and
little experience with computers.
4. EIS and ESS : Executive Information System (EIS) is sometimes referred to as
Executive Support System (ESS). The terms EIS and ESS can be used interchangeable. The difference between the two is that the ESS incorporates
additional capabilities like e-mail, auto responses, etc.
Advantages of an EIS :
1. Easy Access : Executives can have easy access to corporate data. DSS provides
executives with access to Financial data, Marketing and Sales information, Human
Resources information, Manufacturing data and Competitive / Strategic
information. Electronic mail, access to external news and databases, Word
processing, Spreadsheet and automated filing capabilities are also common in
business EIS.
2. Easy navigation : An EIS is easy to navigate since Managers can identify broad
strategic issues, and then explore the information to find the root causes of those
issues.
3. Report Generation : Reports can be generated by the Executives directly, who
can now examine departments without interfering with the operations of the
Company. Graphs can be created automatically.
4. Simulation Facility : Executives can set up different scenarios or simulations. The
ability to manipulate data, to project what if outcomes and to work with modeling
tools within the system are available in EIS.
5. Cost vs. Benefits : EIS is expensive to develop and maintain. But enhanced toplevel decisions making is the benefit that balances out the systems associated
costs.
Various decisions undertaken through EIS :
(a) Strategic Planning involves determining the general, long-range direction of the
Firm. It addresses the general concerns of the Firm. The CEO is ultimately
responsible for the development of strategic plans.

[Chapter # 1] Inform ation System Concepts O

Q&A-6.65

(b) Tactical Planning refers to the how, when, where and what of the issues
involved in carrying out the Strategic Plan. Executives will not normally be
concerned with tactical details, but about general tactics, e.g. the Vice President
(Finance) addresses how the Firm can best achieve a balance between Debt and
Equity financing, the Marketing Vice-President considers which classes of products
should be produced to be successful in the market.
(c) Fire Fighting Activities address the major problems that requires the attentions
of Executive Level Manager, e.g. (i) if Company is involved in a big lawsuit
threatening its financial solvency, (ii) damage to a major facility, (iii) announcement
of an important product by a competitor, (iv) a strike, (v) a sharp reversal of the
economy, etc.
(d) Control : Executive Management also needs to exert some general control over
the Organisation. For example, if a Strategic Plan calls for a 30% increase in
profitability, constructive feedback from departments initiating such action is
required to ensure accomplishment of that objective. Executives should periodically
review key performance data to see how they compare against planned amounts.
Purposes of an EIS :
1. Managerial Learning :

An EIS serves many purposes. The primary purpose of an Executive


Information System is to support managerial learning about an organization,
its work processes, and its interaction with the external environm ent.

Informed managers can ask better questions and make better decisions.
2. Timeliness :

EIS allows timely access to information. Timely access also influences


learning. W hen a manager obtains the answer to a question, that answer
typically sparks other related questions in the managers mind. If those
questions can be posed immediately, and the next answer retrieved, the
learning cycle continues unbroken.

Using traditional methods, by the time the answer is produced, the context of
the question may be lost, and the learning cycle will not continue.
3. Management Attention :

An EIS has a powerful ability to direct management attention to specific areas


of the organization or specific business problems. Some managers see this
as an opportunity to discipline subordinates.

Sometimes misaligned reporting system can result in inordinate management


attention to things that are not so important. An EIS system can provide
information that is actually important and represents a balanced view of the
organisations objectives.

Q&A-6.66

Solved Scanner

CA Final Gr. II Paper - 6

2006 - May [1] {C} (c) Discuss the limitations of the Management Information system.
(5 marks)
Answer :
In present scenario, MIS is only in adolescent stages. The following are the limitation
of MIS :1. One slacking sub-system would throw the entire integrated MIS out of gear.
2. The success of MIS hinges on the quality of software for which there is scarcity of
the brainware, i.e., systems analysts, system designers and programmers.
3. Budgeting of MIS is extremely difficult.
4. MIS is not a substitute for effective management. It means that it cannot replace
managerial judgement in making decisions in different functional areas. It is merely
an important tool in the hands of executives for decision-making and problem
solving.
5. MIS is inherently fragile, highly sensitive and requires continuous monitoring.
6. It cannot provide tailor-made information packages suitable for the purpose of
every type of decision made by executives.
7. Its effectiveness decreases due to frequent changes in top management
organisational structure and operational team.
8. Its effectiveness is reduced in those organisations where the culture of hoarding
information and not sharing with others hold.
9. It is less useful for making non-programmed decisions-Such type of decisions are
not of routine type and thus they require information, which may not be available
from existing MIS to executives.
10. It takes into account mainly quantitative factor; thus it ignores the non-quantitative
factors like morale, attitude of members of the organisation, which have an
important bearing on the decision-making process of executives.
11. Quality of the outputs of MIS is basically governed by the quality of inputs and
processes.
12. MIS may not have requisite flexibility to quickly update itself with changing needs
of time, especially in a fast changing and complex environment.
2006 - May [2] (a) W hat do you mean by Information ? Describe the important
characteristics of information, which m akesituse ful to the organisation.
(10 marks)
(b) Information is necessary to executive for performing the function of planning.
Substantiate the above statement with regard to information requirements of a
marketing system.
(5 marks)
(c) Describe various decisions which can be made with the help of Financial
Information system.
(5 marks)

[Chapter # 1] Inform ation System Concepts O

Q&A-6.67

Answer :
Definition of Information

Information is a necessary and vital input in any decision-making process in an


organisation. However, it is not available in ready form; rather it has to be
generated from data which acts as a raw material that needs some processing.
Figure depicts the generation of information.

Therefore, information in its unprocessed form is called data, which is generated


as a by-product of transactions taking place in the organisation. Information, on the
other hand, is processed data and has an element of surprise.
Information reduces uncertainty and triggers action.
Davis and Olson have defined Information as data that has been processed into
a form that is meaningful to the recipient and is of real or perceived value in current
or prospective actions or decisions.

Types of Information
Broadly speaking, information could be classified on the basis of the purpose for which
it is utillised, into three main categories; namely:
(i) Strategic information,
(ii) Tactical information, and
(iii) Operational information.
1. Strategic Information

Strategic information is required by the managers at the strategic level of


managem ent for the formulation of organisational strategies.

This relates to long-term planning policies of the organization as a whole. e.g.,


information pertaining to new technologies, new products, competitors, etc.
2. Tactical Information

Information in this category is used in short-term planning and is of use at


management control level. e.g., for sales analyses and forecasts, production
resource requirements, annual financial statements, etc.

This type of information is generally based on data arising from current


activities of the organisation.

However, some of the tactical information, such as competitor information,


may arise from sources external to the organisation.

Q&A-6.68

Solved Scanner

CA Final Gr. II Paper - 6

3.

Operational Information

Operational information applies to short periods which may vary from an hour
to a few days.

It is generally used by decision-makers at the operational level. It is often


required for taking immediate action. Examples of operational information may
include current stock-in-hand, work-in-progress levels, outstanding orders
from customers, etc.

The source of such an information is usually current activity data.


Organizational Information (classification of information)

Information is everywhere in an organization. W hen addressing a significant


business issue, employees must be able to obtain and analyze all the relevant
information so they can make the best decision possible.

Organizational information comes at different levels and in different formats and


granularities. Information granularity refers to the extent of detail within the
information (fine and detailed or coarse and abstract.

Employees must be able to correlate the different levels, formats, and granularities
of information when making decision.

Successfully collecting, compiling, sorting, and finally analyzing information from


multiple level, in varied formats, exhibiting different granularity can provide
tremendous insight into how an organization is performing.

Taking a hard look at organizational information can yield exciting and unexpected
results such as potential new markets, new ways of reaching customers, and even
new ways of doing business.
Table showing Levels, Formats, and Granularities of Organizational information
Information
Types

Information
Levels

Information
Formats

Range

Examples

Individual

Individual knowledge, goals, and strategies

Department

Departmental goals, revenues,


processes, and strategies

Enterprise

Enterprise wide revenues expenses, processes,


and strategies

Document

Letters, memos, faxes, e-mails, reports, marketing


materials, and training materials

Presentation

Product, strategy, process, financial, customer, and


competitor presentations

expenses,

[Chapter # 1] Inform ation System Concepts O

Information
Granularities

Q&A-6.69

Spreadsheet

Sales, marketing, industry, financial, competitor,


customer, and order spreadsheets

Database

Customer, employee, sales, order, supplier, and


manufacturer databases

Detail (Fine)

Reports for each salesperson, product, and part

Summary

Reports for all sales personnel, all products, and all


parts

Aggregate
(Coarse)

Product, strategy, process, financial, customer, and


competitor presentations

After understanding the different levels, formats, and granularities of information,


it is important to look at a few additional characteristics that help determine the value
of information. These characteristics include transactional, analytical, timeliness, and
quality.
The value of Transactional and Analytical Information

Transactional information encompasses all of the information contained within a


single business process or unit of work, and its primary purpose is to support the
performing of daily operational tasks.

Examples of transactional information are withdrawing cash from an ATM, making


ah airline reservation, or purchasing stocks.

Organizations capture and store transactional information in databases, and they


use it when performing operational tasks and repetitive decisions such as
analyzing daily sales reports and production schedules to determine how much
inventory to carry.

Analytical information encompasses all organizational information, and its primary


purpose is to support the performing of managerial analysis tasks. Analytical
information includes transactional information along with other information such as
market and industry information.

Examples of analytical information include trends, sales, product statistics, and


future growth projections.

Analytical information is used when making important ad hoc decisions such as


whether the organization should build a new manufacturing plant or hire additional
sales personnel.
The Value of Timely Information

The need for timely information can change for each business decision. Some
decisions require weekly or monthly information while other decisions require daily
information.

Q&A-6.70

Solved Scanner

CA Final Gr. II Paper - 6

Timeliness is an aspect of information that depends on the situation. In some


industries, information that is a few days or weeks old can be relevant while in
other industries information that is a few minutes old can be almost worthless.
The Value of Quality Information

One of the major International banks operating in India, serves millions of


customers from its many core systems, each with its won database.

The databases maintain information and provide users with easy access to the
stored information. Unfortunately, the company failed to develop informationcapturing standards, which led to inconsistent organizational information.

For example, one system had a field to capture e-mail addresses while another
system did not. Duplicate customer information among the different systems was
another major issue, and the company continually found itself sending conflicting
or competing messages to customers from different operations of the bank.

A customer could also have multiple accounts within the company, one
representing a life insurance policy and one representing a credit card. The Bank
had no way to identify that the two different customer accounts were for the same
customer.

W FS had to solve its information quality problems immediately if it was to remain


competitive.

The company purchased NADIS (Name & Address Data Integrity Software), a
software solution that filters customer information, highlighting missing, inaccurate,
and redundant information. Customer service ratings are on the rise for W FS now
that the company can operate its business with a single and comprehensive view
of each one of its customers.
The Five Common Characteristics of High Quality Information
Characteristics of High Quality Information
Accuracy

Are all the values correct? For example, is the name spelled
correctly? Is the dollar am ount recorded properly?

Completeness Are any of the values missing? For example, is the address
complete including street, city, state, and zip code?
Consistency

Is aggregate or summary information in agreement with detailed


information? For example, do all total fields equal the true total of
the individual fields?

Uniqueness

Is each transaction, entity, and event represented only once in the


information? For example, are there any duplicate customers?

[Chapter # 1] Inform ation System Concepts O


Timeliness

Q&A-6.71

Is the information current with respect to the business


requirements? For example, is information updated weekly, daily or
hourly?

The costs of poor information


Using the wrong information can lead to making the wrong decision. Making the wrong
decision can cost time, money, and even reputations. Every business decision is only
as good as the information used to make the decision. Bad information can cause
serious business ramifications such as :

Inability to accurately track customers, which directly affects strategic initiatives


such as Customer Relationship Management (CRM) and Supply Chain
Managem ent (SCM).

Difficulty identifying the organizations most valuable customers.

Inability to identify selling opportunities and wasted revenue from marketing to


nonexisting customers and nondeliverable m ail.

Difficulty tracking revenue because of inaccurate invoices.

Inability to build strong relationships with customers - which increases their buyer
power.
The benefits of goods information
High quality information can significantly improve the chances of making a good
decision and directly increase an organizations bottom line.
The success of the organisation depends on appreciating and leveraging the true value
of timely and high quality information.
(b) The planning information requirements of executives can be categorised into
three broad categories.
1.

Environment
information :

It comprises of the following :


(a) Government policies: Information about concessions/benefits, government policies in respect of tax
concessions or any other aspects, which may be useful
to the organisation in the future period.
(b) Factors of production: Information related with
sources, cost, location, availability, accessability
and productivity of the major factors of production viz., (i)
labour, (ii) materials and parts and (iii) capital.
(c) Technological environment: Forecast of any
technology changes in the industry and the
probable effect of it on the firm.
(d) Economic trends: It includes information relating
to economic indicators like consumer disposal income,

Q&A-6.72

Solved Scanner

CA Final Gr. II Paper - 6

employment, productivity, capital investment etc. Such


information is valuable for those firms specially whose
output is a function of these important variables.
2.

Competitive
information :

It includes the following information


(a) Industry demand: Demand forecast of the
industry in respect of the product manufactured and in
the area in which the firm would be operating.
(b) Firm demand: Assessment of the firms product
demand in the specified market. It also includes an
assessment of firms capability to meet firms
dem and.
(c) The competitive data: Data of competing firms for
forecasting demand and making decision and plans
to achieve the forecast.

3.

Internal
information :

It usually includes information concerning organisations (a)


sales forecast, (b) financial plan/budget, (c) supply factors,
and (d) policies, which are vital for subsidiary planning at all
levels in the organisation.

(c) Please refer 2003 - Nov [2] (a) on Page no.47


2006 - May [3] (b) Describe various software tools used in Decision support system.
(5 marks)
Answer :
The tools of decision support systems : The tools of decision support include a
variety of software supporting database query, modeling, data analysis, and display. A
comprehensive tool kit for DSS would include software supporting these application
areas. Examples of software tools falling into these four categories are given in Table
below:
1.

Data-based
Software

(i)
(ii)
(iii)
(iv)
(v)
(vi)
(vii)

MS-ACCESS
Dbase IV
FOCUS
NOMAD II
RAMIS
FAXPRO
R : base 5000

2.

Model Based
Software

(i)
(ii)
(iii)
(iv)
(v)
(vi)
(vii)

EXCEL
Foresight
IFPS
Lotus 1-2-3
Model
Multiplan
Omncalc

3.

Statistical
Software

(i)
(ii)
(iii)

SAS
SPSS
TSAM

4.

Display-Based
Software

(i)
(ii)
(iii)

Chart Master
SASGRAPH
TELLAGRAF

[Chapter # 1] Inform ation System Concepts O


(viii)
(ix)

Q&A-6.73

ORACLE
SQL

Software tools for decision support systems (DSS)


Database Languages : Tools supporting database query and report generation use
mainframe, minicomputer, and microcomputer-based databases. FOCUS, RAMIS, and
NOMAD II, for example, are mainframe-based languages supporting database query,
report generation, and simple analysis.
Model-Based Decision Support Software : Model-based analysis tools such as
spreadsheet software enable managers to design models that incorporate business
rules and assumptions. Microcomputer-based spreadsheet programs such as
Lotussuits, and, Excel all support model building and "what if ?" types of analysis.
Mainframe-based spreadsheets such as Megacalc and Omnicalc fulfill the same
purpose. Modeling tools like IFPS and Model are designed to support financial modeling
and analysis.
Tools for Statistics and Manipulation : Statistical analysis software such as SAS and
SPSS supports market researchers, operations research analysts, and other
professionals using statistical analysis functions. Because of the need for increased
number crunching" capabilities, this types of software usually runs on mainframe
computers. Microcomputer-based statistical packages are available as well. For
example, the micro-based version of SPSS has about the same capabilities as the
mainframe version, but it is much slower.
Display-Based Decision Support Software : The final category of decision support
software is display-based software. Graphic displays of output generated from MSExcel spreadsheets.
Integrated tools : One issue concerning decision support system is the need for
integrated tools. Integrated tools provide the ability to generate, manipulate, and
statistically analyse data within a single software package. Individual tools supporting
isolated decision support functions such as database query or modeling exist, but these
tools are not always integrated with each other.
An integrated tools can transfer data from a spreadsheet into a graphics program
or from a databases into a statistics programs.
2006 - Nov [2] (a) State the factors to be considered for designing an effective
Management Information system.
(10 marks)
(b) Enumerate various information which are required for sales support and sales
analysis.
(10 marks)
Answer:
(a) Please refer 1999 - Nov [1] {C} (a) on page no. 29

Q&A-6.74

Solved Scanner

CA Final Gr. II Paper - 6

(b) Information requirements for sales support and sales analysis are
enumerated below:
(i) Sales support : A specialized sales support information system must provide
the following information to sales personnel :
1. Products descriptions and perform ance specifications.
2. Product Prices.
3. Quantity discounts and other product discount information.
4. Sales incentives for salespersons.
5. Sales promotions.
6. Financing plans for customers.
7. The strengths and weaknesses of competitors products.
8. The histories of customers relations with the company.
9. Sales policies and procedures established by the company.
10. Products that have not yet been introduced.
11. Products inventory levels.
12. Buying habits of consumers.
These and other information come from a variety of sources. Product
performance specifications, for example, may come from the engineering
department.
(ii) Sales Analysis :
Challenger with the competitors in the market, the sales analysis is a very vital
activity. It must provide the following information.
1. Sales trends (product-wise)
2. Product wise profitability
3. Region, branch wise sales performance
4. Sales person wise performance
C
The above information is derived from Sales Transaction Processing System as
well as from other system like Financial Accounting System and HR Management
System etc.
C
The majority of information comes from actual sales transactions and is contained
on sales invoices. To fully support the sales analysis system, invoices should
contain information about product type product quality, price discount terms,
customer identity and type, sales region, and salesperson.
C
Information from other sources should also be included in the sales reports.
Specifically, the sales reports must contain information about the profitability of
products, product lines, sales territories, and individual sales persons. Profitability
reporting requires information about product administrative and selling costs.

[Chapter # 1] Inform ation System Concepts O

Q&A-6.75

2007 - May [3] (b) Decision support systems are widely used as part of an
Organizations Accounting Information system. Give examples to support this
statements.
(10 marks)

Answer:
C

Decision Support System are widely used as part of an organisations AIS. The
complexity and nature of decision support systems vary from organization to
organization.
Many are developed in-house using either a general type of decision support
program or a spreadsheet program to solve specific problems.

Below are the examples of DSS in Accounting includes:

1.

General
Decision
Support
System:

C
C
2.

Capital
Budgeting
System :

C
C

C
C

Such types of decision support systems are a decisionmakers tools that are used to input the data and answer
questions about a specific problem domain to make use of this
type of decision support system.
An example is a program called Expert Choice. This
program supports a variety of problems requiring decision.
The user works interactively with the computer to develop a
hierarchical model of the decision problem.
The decision support system them asks the users to
compare decision variables with each other.
Expert Choice analyses investment judgments and
presents the decision maker with the best alternative.
Companies require new tools to evaluate high technology
investment decisions.
Decision makers
need
to
supplement
analytical
techniques with decision support tools that consider some
benefits of new technology. One decision support system
designed to support decisions about investments in automated
manufacturing technology that is Auto Man, which allows
decision makers to consider financial, non-financial,
quantitative, and qualitative factors in their decision-making
processes.
Using this decision support system, accountants,
managers, and engineers identity and prioritize these factors.
They can then evaluate up to seven investment

Q&A-6.76

Solved Scanner

CA Final Gr. II Paper - 6

alternatives at once.
3.

C
Cost
Accounting
System :
C

C
C

4.

Budget
Variance
Analysis
System :

The health care industry is well-known for its cost


complexity. Managing cost in this industry requires
controlling costs of supplies, expensive m achinery,
technology, and a variety of personnel.
Cost
Accounting
a pplications
he lp
health
care
organisation calculate product costs for individual procedures
or services.
DSS can accumulate these product costs to calculate total
costs per patient.
Combining cost accounting DSS and Productivity system
applications allows mangers to measure the effectiveness
of specific operation processes to improve its management
decision-making.
Financial institutions rely heavily on their budgeting
systems for controlling costs and evaluating managerial
performance, DSS allows comptrollers to graph, view, analyse,
and annotate budget variances, as well as create additional
one-and five year budget projections using the forecasting
tools provided in the system .
The decision support system thus helps the comptrollers
create and control budgets for the cost-center managers
reporting to them.

2007 - May [5] (c) What is Transaction processing cycle ?Discuss briefly four common
cycles of a business activity.
(5 marks)
Answer:
Transaction Processing System (TPS)
1. TPS is an Information System (IS) that manipulates data from business
transactions.
2. These system accumulate information in databases that form the foundation for
higher-level systems say, MIS, DSS, and EIS.
3. Typically, a TPS involves the following activities :
(i) Capturing data to organize in files of databases.
(ii) Processing of files / databases using applications software.
(iii) Current activity processing e.g., stock warnings, issue notes, replenishment
orders.
(iv) Report processing e.g., material reports scrap reports, pareto analysis.

[Chapter # 1] Inform ation System Concepts O

4.

Q&A-6.77

(v) Inquiry/query processing in relation to orders processing, Stock turnover,


Orders outstanding etc.
These systems also manage supply-chin. For example purchasing an item at a
Reliance fresh store generates more than a cash register receipt, it also
automatically sends a restocking order to the appropriate supplier.

5.

A transaction processing system may follow periodic data preparation and batch
processing (As in payroll application) or on-line processing (as in inventory control
application).
6. People who participate in Transaction processing system (TPS) usually are not in
a position to take any managem ent decision.
Components of TPS

The principal components of a transaction processing system include inputs


processing, storage, and outputs. These components or elements are part of both
manual and computerized systems.
Inputs : Source documents, such as customer orders, sales slips, invoices, purchase
orders, and employee time cared, are the physical evidence of inputs into the
transaction processing system. They serve several purposes.:
1. Capture data.
2. Facilitate operations by communicating data and authorising another operation in
the process.
3. Standardize operations by indicating what data require recording and what actions
need to be taken.
4. Provide a permanent file for future analysis, if the documents are retained.
Source documents are typically forms carefully designed for ease of use and accurate
data capture.
Processing : Processing involves the use of journals and registers to provide a
permanent and chronological record of inputs. The entries are done either by hand in
simple manual systems (journalised) or by a data entry operator using a PC. Journals
are used to record financial accounting transacts, and registers are used to record other
types of data not directly related to accounting.

Q&A-6.78

Solved Scanner

CA Final Gr. II Paper - 6

Journals are used to provide a chronological record of financial transactions. It is,


theoretically possible, but not often practicable, to use the two column general ledger
as the only book of original entry. However, to effect a division and saving of labour,
special journals with special analysis columns are used to record similar and recurring
transactions. Some of the more common special journals that may by kept are as
follows:
1. Sales journal: used to summarise sales made on account.
2. Purchase journal: used to summarize purchase made on account.
3. Cash receipts journal: used to summarize receipts of cash.
4. Cash disbursements journal: used to summarise disbursements of cash.
Storage : Ledgers and files provide storage of data in both manual and computersied
systems. The general ledger, the accounts vouchers payable ledger, and the accounts
receivable ledger are the records of financial account. They provide summaries of a
firms financial accounting transactions. All accounting transactions must be reflected
in the general ledger. Adebit-credit entry is input for every transaction.
Computer storage : A file is an organized collection of data. There are several types
of files. A transaction file is a collection of transaction input data. Transaction files
usually contain data that are of temporary rather than perm anent interest. By contrast,
a master file contains data that are of a more permanent or continuing interest. To
illustrate this difference, consider the posting of sales on accounting to the accounts
receivable ledger. Because a sales journal is a chronological record of sales-on-account
transactions, it may be called a transaction file. The transaction file consists of raw data
concerning sales to customers. Although there may be several sales to the same
customer, this will not be known until the transaction data are processed. The process
of posting sales to the accounts receivable ledger summarizes sales to an individual
customer.
Computer Processing : W hen computer are use for processing, two different modes
of processing accounting transactions are possible. These modes are batch processing
and direct processing. Batch processing is conceptually very similar to a traditional
manual accounting system. Batches of transaction are accumulated as a transaction
file. Transaction files are printed to provide documentation of inputs to the accounting
system. Transaction files are subsequently posted to ledgers by computer programs.
Processing converts data information : Management is more interested in sum mary
data such as total sales and total account balance than in the details of a particular
sales transaction. Management thus has a permanent interest in the information that
is contained in the accounts receivable master file. In contrast, managements interest
in transaction files is temporary.

[Chapter # 1] Inform ation System Concepts O

Q&A-6.79

Outputs : There is a wide variety of outputs from a transaction processing system. Any
document generated in the system is an output. Some documents are both output and
input (e.g.,a customer invoice is an output from the order-entry application system and
also an input document to the customer). Other common outputs of a transaction
processing system are the trial balance, financial reports, operational reports, pay
cheques, bills of lading, and voucher cheques (payments to vendors).
The trial balance lists the balance of all the accounts in the general ledger and tests the
accuracy of the record keeping. Thus, it is fundamental to financial control and
preparation of financial statements.
Financial reports summarise the results of transaction processing and express
these results in accordance with the principles of financial reporting. Two common
financial reports are the balance sheet and the income statement. In addition to these
two reports, a wide variety of financial reports can be prepared to suit the needs of
management and others. Operational reports summarize the results of transaction
processing in a statistical or comparative format. Reports summarizing goods received,
goods ordered, customer orders received, and other such activities are essential to the
operation of a firm. The nature and content of such reports depend on the nature of a
firm and its transaction processing activities.
Transaction processing requires the management of accounting data as they flow
through AIS. When planning a new system, the developers usually start by designing
the outputs from the system. These outputs, and especially managerial reports, then
become the goals of the AIS and therefore provide a focus for the prerequisite tasks of
data collection and data processing. Poorly designed reports can harm the value of an
AIS. Sometimes reports include too much data. To avoid overloading m anagers with
data, an AIS should incorporate elements of good report design.
Outputs, then, drive the inputs to an AIS. The fundamental instrument for collecting
data in a typical AIS is the source document. Source documents should be easy to read,
easy to understand, and serve to collect and distribute information as well as establish
authenticity or authorisation.
For transaction processing system to gather and process data efficiently,
accounting data are often coded. AlSs can use codes to identify accounting information
uniquely, to compress data, to classify transactions in accounts, and to convey special
meanings.
2007 - Nov [1] {C} (c) Discuss any five benefits which are attained by implementing a
computerised model for making decision.
(5 marks)
Answer:
Please refer 2003 - May [2] (a) on page no. 42

Q&A-6.80

Solved Scanner

CA Final Gr. II Paper - 6

2007 - Nov [2] (b) System analysts develop various categories of information systems
to meet a variety of business needs. Discuss any three such systems briefly.
(10 marks)
Answer:
Please refer 2001 - Nov [1] {C} (a) on page no. 35
2007 - Nov [6] (c) Briefly explain the principles to guide the design of measures and
indicators to be included in EIS.
(5 marks)
Answer:
Executive Information System
Please refer 2001 - May [2] (b) on page no.34
Roles of EIS :C
Functions deal primarily with data about the external environment and data that
come from intormal sources, they are usually less reliant on direct contact with
information technology then other types of managers.
C
W hen information from their companys computers is needed, many chief
executive officers make their subordinates retrieve that information. Because
executive information needs are more ambiguous than those of other levels of
management, computers have historically been less useful to executives.
C
Many executives have little hands-on experience with computers and do not fully
appreciate now information technology can improve their personal productivity and
decision making skills.
C
Studies EIS implementation show that thousands of companies have implemented
executive information systems. Usually, the executive information systems provide
executives with access to financial data, marketing and sales information, human
resources information, manufacturing data, and competitive/ strategic information.
C
Electronic mail, access to external news and databases, word processing,
spreadsheet, and automated filing capabilities are also common in business
executive information systems.
C
W hile it is often expensive to develop and maintain an EIS, many organizations
feel that enhanced top level decision making is a benefit that more than balances
out any costs associated with the system.
Characteristics of an EIS:Please refer 2001 - May [2] (b) on page no. 34
2008 - May [2] (b) Describe the main prerequisites of a MIS which makes it an effective
tool. Explain the major constraints in operating it.
(10 marks)
Answer :
Please refer 1999 - Nov [1] {C} (a) on page no. 29

[Chapter # 1] Inform ation System Concepts O

Q&A-6.81

2008 - May [4] (b) Briefly discuss four basic components of Decision Support System.
(5 marks)
Answer:
Components of a DSS : A decision support system has four basic components:
(1) One or more databases,
(2) The user,
(3) The model base
(4) A planning language, and
Decision support system

1.

2.

Database/s - DSS include one or more databases. The database is comm on to all
users of the system but is independent of the programs which use the data. It
contains routine and non routine data from both internal as well as external sources
an example being, data about economic/market conditions, level of competition
etc.
It the database is too large for manual searching then a computer supported
approach may be worthwhile.
User - User of a DSS is the manager with semi-structured or unstructured problem
to solve. The users must have a thorough understanding of the problem and the
factors to be considered for solving that problem. It is not necessary that user of
DSS should have thorough knowledge of computers/computer programming for
using DSS for solving a problem as most of the DSS models use easy-to-use
software which utilize natural language.

Q&A-6.82

Solved Scanner

CA Final Gr. II Paper - 6

3.

Model base - The planning language in DSS allows the users to maintain a dialog
with the model base. The model base is the brain of the DSS. Since it, performs
data manipulation/processing and other specific functions ranging from
mathematical calculations to financial computation.
4. Planning languages - There are 2 types of planning languages that are commonly
used in DSS.
C
General - purpose planning languages allow users to perform many routine
tests for example spreadsheets, they allow budgeting, forecasting and other
worksheets oriented problems.
C
Special - purpose planning languages are used for specified purposes such
as PERT, CPM, SAS, SPSS and Minitab.
2008 - May [5] (c) Personnel information system deals with flow of information relating
to people Explain.
(5 marks)
Answer:
Please refer 2000 - Nov [1] {C} (a) on page no. 30

CA Final Gr. II (New Course)


S HORT N OTES
2010 - Nov [7] Write short notes on the following :
(c) Benefits of Expert Systems
Answer :
Please refer 2004 - May [7] (d) on page no. 19

(4 marks)

2011 - May [7] Write short notes on the following :


(a) Business applications of Expert Systems for Management Support Systems.
(4 marks)
Answer :
Business applications of Expert Systems for Management Support Systems are given
as follows:
(i) Finance: It provides tax advice and assistance, helping with credit authorization
decisions, selecting forecasting models, providing investment advice.
(ii) Marketing: It helps establishing sales quotas, responding to customer inquiries,
referring problems to telemarketing centers, assisting with marketing timing
decisions, determ ining discount policies.
(iii) Manufacturing: It helps in determining whether a process is running correctly,
analyzing quality and providing corrective measures, maintaining facilities,

[Chapter # 1] Inform ation System Concepts O

Q&A-6.83

scheduling job-shop tasks, selecting transportation routes, assisting with product


design and faculty layouts.
(iv) Personnel: It is useful in assessing applicant qualifications, giving employees
assisting at filling out forms.
(v) General Business: It helps in assisting with project proposals, recommending
acquisition strategies, educating trainees, evaluating performance.
2011 - Nov [7] Write Short Notes on following :
(a) Objectives of an Operating System

(4 marks)

Answer :
Objectives of an Operating System: An operating system (OS) is a program that
controls the execution of an application program and acts as an interface between the
user of a computer and computer hardware. The m ain purpose of an OS is to provide
an environment in which a user can execute programs in a convenient and efficient
manner. An operating system is an im portant part of almost every computer system. It
is considered to be the backbone of a computer, managing both software and hardware
resources. OS is responsible for everything from the control and allocation of memory
to recognizing input from external devices and transmitting output to computer displays.
It also manages files on com puter hard drives and control peripherals, like printers and
scanners.
Major objectives/functions of an operating system are as follows:
1. Scheduling of Jobs: It can determine the sequence in which jobs are executed,
using priorities established.
2. Managing Hardware and Software Resources: It can first cause the users
application program to be executed by loading it into primary storage and then
cause the various hardware units to perform as specified by the application.
3. Maintaining System Security: It may require users to enter a password - a group
of characters that identifies users as being authorized to have access to the
system.
4. Enabling Multiple User Resource Sharing: It can handle the scheduling and
execution of the application programs for many users at the same time, a feature
called multiprogramming.
5. Handling Interrupts: An interrupt is a technique used by the OS to temporarily
suspend the processing of one program in order to allow another program to be
executed. Interrupts are issued when a program requests an operation that does
not require the CPU, such as input or output, or when the program exceeds some
predeterm ined time limit.

Q&A-6.84
6.

Solved Scanner

CA Final Gr. II Paper - 6

Maintaining Usage Records: It can keep track of the amount of time used by
each user for each system unit - the CPU, secondary storage, and input and output
devices. Such information is usually maintained for the purpose of charging users
departments for their use of the organizations computing resources.

2012 - Nov [7] Write short notes on the following :


(c) Limitations of MIS.
Answer :
Please refer 2006 - May [1] {C} (c) on page no. 66

(4 marks)

D ESCRIPTIVE Q UESTIONS
2008 - Nov [2] (b) What is Decision Support System? Briefly explain three
characteristics of Decision Support System.
(5 marks)
(c) Explain Executive Information System (EIS). What purpose does it serve?
(5 marks)
Answer :
(b) Please refer 2002 - Nov [7] (c) on page no. 16
(c) Please refer 2005 - Nov [3] (a) on page no. 64
2008 - Nov [3] (a) What do you understand by classifications of Information? Explain
different classifications of information.
(10 marks)
Answer :
Please refer 2006 - May [2] (a) on page no. 66
2009 - Nov [2] (a) Identify and justify the type of each one of the following systems
based on how they perform within an environment and/or certainty/uncertainty :
(i) Marketing system
(ii) Communication system
(iii) Manufacturing system
(iv) Pricing system
(v) Hardware-Software system.
(5 marks)
Answer :
System

System Type

Justification

[Chapter # 1] Inform ation System Concepts O


(i)

Mar keti n Open System C


g system
C

(ii)

Commun Open System C


ication
system
C

(iii)

Manufact Closed
-uring
System
System

C
(iv)

Pricing
System

Probabilistic
and Open
System

(v) Hardw are


Soft- ware
System

Closed Deter- C
ministic
System
C

Q&A-6.85

The marketing system plays a pivotal role in the


running of a business in the competitive
environment.
The objective of the system is to maximize
customer satisfaction by providing a free
interactive environment.
The system takes input/ feedbacks and
facilitates the outcomes as products of the
company and to create new customers.
The communication system in an organization
is a point of contact to balance the external
influence and render its services to the
customers.
The system interacts freely with its environment
by taking input and returning output.
Close system is in place to meet a particular
objective. It neither interacts with the
environment nor changes with the change in
the environment.
A manufacturing unit is completely isolated
from its environment for its operation.
Pricing system has a probable behavior and
interacts freely with its environment by taking
inputs and returning outputs,.
The pricing system is a dynamic one which
influences the form of profit and goodwill of an
organization.
The interaction among the various parts of the
system is known with certainty and it does not
interact with the environment and does not
change with the change in the environm ent.
In this case the requirements of the hardware
and software inventory are known with
certainty.
The operational state of these systems is
predictable.

2010 - May [4] (c) Give some important advantages of Information System in business.

Q&A-6.86

Solved Scanner

CA Final Gr. II Paper - 6


(5 marks)

Answer :
Importance of information system in Management Process/ Business Process :
1. Information is required for proper handling of complex managerial problems and
decision making. Innovative ideas for solving critical problems may come out from
good information system.
2. Information makes the Managers realise the need for taking decisions. Information
is a vital input for Managers just like men, money, materials, machinery and
methods.
3. Information helps to-(a) recognise and define the problem, (b) develop alternative
courses of action, and (c) select the best course of action, (d) get have a better
view of the decision situations, and (e) reduce the complexity and uncertainty in
decision making.
4. Information significantly determines the effectiveness of both the final decision,
and the process of decision-making.
5.

6.
7.

Information systems helps take right decision at the right time. So, based on welldesigned information system, an organization will gain edge in the competitive
environment.
Knowledge gathered though information system may be utilized by managers in
unusual situations.
If Information System is viewed as a process, it can be integrated to formulate a
strategy of action or operation.

2010 - May [5] (c) Briefly describe any three of the characteristics of the types of
information used in Executive Decision making.
(5 marks)
Answer :
Please refer 2001 - May [2] (b) on page no. 34
2010 - Nov [5] (b) What do you understand from the term database ? How is it
implem ented in three different levels ?
(4 marks)
Answer :
Database : Database can be defined as a Super-file which consolidates data records
formerly stored in many data files. The data in a database is organized in such a way
that access to the data is improved and redundancy is reduced.
Implementation of Databases : There are the levels at which database can be
implemented :(i) Physical Level : It involves the implementation of the database on the hard disk.
The management of storage and access is controlled by operating system.

[Chapter # 1] Inform ation System Concepts O

Q&A-6.87

(ii)

Logical Level : It is designed by professional programs, who have complete


knowledge of DBMS. The storage is logically divided into various tables having
techniques for defining relationships with indexes.
(iii) External Level : The logical level defines schema, which is divided into smaller
units known as sub-schemas and given to the managers, each sub-schema
containing all relevant data needed by one manager.
2011 - May [3] (b) What are the characteristics of Executive Information System ?
(4 marks)
Answer :
Executive Information System is an information system that serves the information
need of top executives. EIS enables its users to extract summary data and model
complex problems without the need to learn complex query languages, statistical
formulas or high computing skills.
EIS is considered as highly user friendly system because it provides a user friendly
graphical reporting system with drill down capabilities. EIS is mainly an advancement
of MIS but it can include the decision support system (DSS) capabilities to solve
complex problems.
Characteristics : (EIS)
1. Easy Access: Executives can have easy access to corporate data. DSS provides
executives with access to Financial data Marketing and Sales information, Human
Resources information, Manufacturing data and Competitive / Strategic
information. Electronic mail, access to external news and databases, word
processing, Spreadsheet and automated filing capabilities are also common in
business EIS.
2. Easy Navigation : An EIS is easy to navigate since Managers can identify broad
strategic issues, and then explore the information to find the root causes of those
issues.
3. Report Generation : Reports can be generated by the Executives directly, who
can now examine departments without interfering with the operations of the
Company. Graphs can be created automatically.
4. Simulation Facility : Executives can set up different scenarios or simulations. The
ability to manipulate data, to project what if outcomes and to work with modeling
tools within the system are available in EIS.
5. Cost vs. Benefits : EIS is expensive to develop and maintain. But enhanced toplevel decision making is the benefit that balances out the systems associated
costs.
2011 - May [5] (b) Enumerate the characteristics of a Computer Based Information
System.
(4 marks)

Q&A-6.88

Solved Scanner

CA Final Gr. II Paper - 6

Answer :
Characteristics of a Computer Based Information System are as follows:
1. The system is designed and developed, according to predetermined objectives.
2. A system has a num ber of interrelated and interdependent subsystems or
components. No subsystem can function in isolation; it depends on other
subsystems for its inputs and provides inputs to others.
3. If one subsystem or component of a system fails, the whole system fails. However,
it depends on how the subsystems are interrelated and interdependent on each
others.
4. The way a subsystems works with another subsystem is called interaction.
Different subsystems interact with each other to achieve the goal of the system.
5. The work done by individual subsystem is integrated to achieve the central goal of
the system.
2011 - Nov [2] (a) Define the term "Information". Discuss various important attributes
that are required for useful and effective information.
(8 marks)

Answer :
Information: Information is the data that have been put into a meaningful and useful
context. It has been defined by Davis and Olson as-Information is data that has been
processed into a form that is meaningful to the recipient and is of real or perceived
value in current or progressive decision. For example, data regarding sales by various
salesman can be merged to provide information regarding total sales through sales
personnel. This information is of vital importance to a marketing manager who is making
plan for future sales.
Attributes of Information: Some important attributes of useful and effective
information are as follows:
C
Availability: If information is not available at the time of need, it is useless. Data
is organized in the form of facts and figures in databases and files from where
various information is derived for useful purpose.
C
Purpose: Information m ust have purposes at the time, it is transmitted to a person
or machine, otherwise it is simple data. Information communicated to people has
a variety of purposes due to variety of activities performed by them in business
organizations. The basic purpose of information is to inform, evaluate, persuade,
and organize.
C
Mode and format: The modes of communicating information to humans are
sensory (through sight, hear, taste, touch and smell) but in business they are either
visual, verbal or in written form. Format of information should be so designed that

[Chapter # 1] Inform ation System Concepts O

Q&A-6.89

it assists in decision making, solving problems, initiating planning, controlling and


searching.
Decay: Value of information normally decays with time and usage and hence, it
should be refreshed from time to time. For example, we access the running score
sheet of a cricket match through Internet sites and this score sheet is continually
refreshed at a fixed interval or based on status of the state.
Rate: The rate of transmission/reception of information may be represented by the
time required to understand a particular situation. Quantitatively, the rate for
humans may be measured by the number of numeric characters transmitted per
minute, such as sales reports from a district office. For machines the rate may be
based on the number of bits of information per character (sign) per unit of time.
Frequency: Frequency with which information is transm itted or received affects
its value. Financial reports prepared weekly may show so little changes that they
have small value, whereas monthly reports may indicate changes big enough to
show problems or trends.
Completeness: The information should be as complete as possible. The classical
ROI or Net Present Value (NPV) models just provide a point estimate and do not
give any indication of the range within which these estimates may vary. Hartzs
model for investment decisions provides information on mean, standard deviation
and the shape of the distribution of ROI and NPV. With this complete information,
the manger is in a better position to decide whether or not to undertake the
venture.
Reliability: It is just not authenticity or correctness of information, but also failure
or success of using information for decision making. If information leads to correct
decision on many occasions, we say the information is reliable.
Validity: It measures the closeness of the information to the purpose which it
purports to serve. For example, some productivity measure may not measure, for
the given situation, what they are supposed to do e.g., the real rise or fall in
productivity. The measure suiting the organization m ay have to be carefully
selected or evolved.
Quality: Quality refers to the correctness of information. Information is likely to
become incorrect due to personal bias. For example, an over-optimistic salesman
may give rather too high estimates of the sales. This problem, however, can be
circumvented by maintaining records of salesmans estimates and actual sales and
deflating or inflating the estimates in the light of this.
Transparency: If information does not reveal directly what we want to know for
decision-making, it is not transparent. For example, total amount of advance does
not give true picture of utilization of fund for decision about future course of action;
rather deposit-advance ratio is perhaps more transparent information in this matter.

Q&A-6.90
C

Solved Scanner

CA Final Gr. II Paper - 6

Value of information: It is defined as difference between the value of the change


in decision behaviour caused by the information and the cost of the information.
In other words, given a set of possible decisions, a decision-maker may select one
on basis of the information at hand. If new information causes a different decision
to be made, the value of the new information is the difference in value between the
outcome of the old decision and that of the new decision, less the cost of obtaining
the information.
Adequacy: Information must be adequate so that the desired actions can be
initiated. Required information should flow on different directions within the
organization and to and from its environment. In addition, the type of information
that flows within the organization or across, should have adequate and relevant
contents.

2012 - May [3] (b) What is Decision Support System? Discuss its characteristics in
brief?
(6 marks)
Answer:
Please refer 2002 Nov [7] (c) on page no.16

2012 - May [5] (c) Discuss the constraints in operating a MIS.


Answer:
Please refer 2006 May [1] (c) on page no.66

(4 marks)

2012 - Nov [2] (b) What is meant by EIS? W hat are its characteristics?
(c) Explain any four features of Electronic Mail.
Answer :(b)
Please refer 2011 - May [3] (b) on page no. 86

(6 marks)
(4 marks)

(c) Electronic M ail


Business organisations are using different type of systems for sending and
receiving the messages electronically. These include, Voice Mail, Fax, E-mail etc.
E-mail has become the most popular electronic message communication system.
It not only helps to transfer messages im mediately but with high reliability. It also
provides an economical method of efficient message communication.
Some features of E-mail are as follows:
On Line Editor
It provides an efficient editor to edit the new or existing mail.
Storage on Magnetic M edia
It helps and facilitates the storage of messages on magnetic media thereby
reducing the space required to store the messages.

[Chapter # 1] Inform ation System Concepts O

Q&A-6.91

Reply and Attachment


This option helps to reply a received mail by writing the message in received
mail itself and documents can be attached.
Forward Mail
This helps in forwarding a received mail to another user. This is due to the
reason that e-mail could be routed to people having direct interest in the
massage with or without changing or/and appending related information to the
message.
2013 - May [2] (c) How does Executive Information System differs from Traditional
Information System?
(4 marks)
2013 - May [3] (b) What is an Expert System? List the properties which an application
should posses to qualify for Expert System development.
(6 marks)

Similarly Asked Questions


No.

Category

Question

Marks

Frequency

Descriptive

W hat are the main pre-requisites of


an effective MIS? Explain them
briefly.
99 - Nov [1] (a) and 05 - May [2] (c)

10, 5

2 Times

Short Note

W rite short notes on Material


requirement planning.
03- May [7] (d) and 06-May [7] (a) (iii)

5, 5

2 Times

Executive Information Systems.


Explain.
01- May [2] (b), 03- Nov [7] (a),
05 - Nov [3] (a), 11 - May [3] (b),
12 - Nov [2] (b)

5, 5,
10, 4, 6

5 Times

Shorts
Notes /
Descriptive

You might also like